PDA

نسخه کامل مشاهده نسخه کامل : اتاق ریاضیات(طرح سؤالات)



صفحه ها : 1 2 [3] 4 5 6 7 8 9 10 11 12 13 14 15 16 17 18 19 20

m_honarmand_j
07-12-2006, 00:46
سلام . يه سوال :
يك قالب پنير سه در سه وجود دارد . موشي از يك گوشه ي پنير شروع به خوردن مي كند . پس از خوردن هر قطعه به قطعه ي مجاور مي رود . دو قطعه مجاورند اگر يك وجه مشترك داشته باشند و پس از خوردن هر قطعه ديگر نمي توان به آنجا بر گشت . آيا مي تواند طوري اين پنير را بخورد كه آخرين قطعه اي كه مي خورد قطعه ي وسطي باشد ؟ :blush: :puke:

جواب . قطعات را به صورت شطرنجي رنگ مي كنيم . بدون آنكه از كليت مسئله كم شود فرض مي كنيم كه رنگ خانه اي كه از آن شروع مي كنيم سياه باشد . در اين صورت داريم كه رنگ خانه ي مركزي سفيد بوده و تعداد خانه هاي سياه برابر 14 و تعداد خانه هاي سفيد 13 است . در هر حركت كه انجام مي دهيم رنگ خانه عوض مي شود در نتيجه به دليل مساوي نبودن تعداد خانه هاي سياه و سفيد ما بايد در آخرين حركت در خانه ي سياه باشيم كه رنگ خانه ي مر كزي سفيد است در نتيجه نمي توان اين كار را انجام داد . :evil: :tongue:

mofidy1
09-12-2006, 19:52
با سلام

دوستان به لینک زیر سری بزنید و نرم افزار Geometer's Sketchpad را که مخصوص تولید تصاویر متحرک ریاضی است، دانلود کنید. از این نرم افزار می توان برای آموزش مفاهیم ریاضی در سطوح مختلف استفاده کرد.

[ برای مشاهده لینک ، لطفا با نام کاربری خود وارد شوید یا ثبت نام کنید ]

دانلود نرم افزار با حجم2.17 مگابایت:

[ برای مشاهده لینک ، لطفا با نام کاربری خود وارد شوید یا ثبت نام کنید ]

mofidy1
09-12-2006, 19:53
با سلام

فرض کنید:


[ برای مشاهده لینک ، لطفا با نام کاربری خود وارد شوید یا ثبت نام کنید ]

ثابت کنید:


[ برای مشاهده لینک ، لطفا با نام کاربری خود وارد شوید یا ثبت نام کنید ]

موفق باشید.

ارسال متن: شنبه 11 آذر 1385

با سلام

برای دیدن راه حل مساله به لینک زیر مراجعه کنید:

[ برای مشاهده لینک ، لطفا با نام کاربری خود وارد شوید یا ثبت نام کنید ]

موفق باشید.

ارسال متن: شنبه 18 آذر 1385

mofidy1
09-12-2006, 20:10
با سلام

فرض کنید:


[ برای مشاهده لینک ، لطفا با نام کاربری خود وارد شوید یا ثبت نام کنید ]

ثابت کنید:


[ برای مشاهده لینک ، لطفا با نام کاربری خود وارد شوید یا ثبت نام کنید ]

موفق باشید.

ارسال متن: شنبه 18 آذر 1385

eh_mn
09-12-2006, 23:53
سلام
9 نقطه در فظاي سه بعدي با مختصات صحيح وجود دارد . ثابت كنيد دوتا از اين نقطه ها هستند كه مختصات نقطه ي وسطشان عددي صحيح است .
سلام
براي هر كدام از مولفه هاي يك نقطه با مولفه هاي صحيح 2 حالت از نظر زوج يا فرد بودن وجود دارد. بنابراين مولفه هاي يك نقطه به 8 حالت مي توانند از لحاظ زوج يا فرد بودن انتخاب شوند. بنابراين از ميان اين 9 نقطه حتما دو نقطه داراي مولفه هايي از نوع يكسان (از نظر زوج يا فرد بودن ) هستند و واضح است كه مختصات وسط اين دو نقطه عددي صحيح است.

rima-maj
10-12-2006, 11:27
makose tabee y=x+[x]

rima-maj
10-12-2006, 11:30
makose tabee y=x+[x] chist

rasool_danesh
10-12-2006, 12:29
با سلام
لطفا به سئوال من پاسخ دهيد و با تشكر

- مثالهايي از توابع F:z---->z بياوريد كه
الف) پوشا باشد ولي يك به يك نباشد
ب) نه پوشا باشد و نه يك به يك
ج) هم پوشا باشد و هم يك به يك
د) يك به يك باشد ولي پوشا نباشد

m_honarmand_j
12-12-2006, 19:19
سلام
دوستان اگه خدا بخواد دارم یه تاپیک جدید درست می کنم و سوالات مربوط به ترکیبیات و ریاضیات گسسته رو اونجا مطرح می کنم . اطلاعات تکمیلی و بعدا می گم . اگه کسی دوستدار این نوع سوالات از این به بعد می تونه منبع خوبی داشته باشه .
اگه کسی نظری داره لطفا بگه . فعلا

m_honarmand_j
12-12-2006, 19:24
سلام eh_mn
راه حل شما در مورد یوال نقاط درست ه . این مسئله با کمک اصل لانه کبوتری حل می شه و این یکی از سوالات ساده در مورد اصل لانه کبو تری است .
خیلی خوشحال شدم دیدم یه نفر یکی از سوالاتی و که گذاشتم حل کرده . موفق باشی .

ali_hp
12-12-2006, 23:52
makose tabee y=x+[x] chist
سلام
[ برای مشاهده لینک ، لطفا با نام کاربری خود وارد شوید یا ثبت نام کنید ]

ali_hp
13-12-2006, 00:05
با سلام
لطفا به سئوال من پاسخ دهيد و با تشكر

- مثالهايي از توابع F:z---->z بياوريد كه
الف) پوشا باشد ولي يك به يك نباشد
ب) نه پوشا باشد و نه يك به يك
ج) هم پوشا باشد و هم يك به يك
د) يك به يك باشد ولي پوشا نباشد
سلام
الف)برای n>0 تعریف کنید f(n)=n-1 و برای n<0 تعریف کنید f(n)=n و اف صفر راهم صفر بگیرید.(این تابع مقدار صفر را دو بار می گیرد)
ب)f را تابع ثابت بگیرید.(مثلا f(n)=1 برای هر n صحیح.)
ج)f(n)=n برای هر n صحیح.
د)برای n>0 تعریف کنید f(n)=n+1 و برای بقیه n های صحیح(منفی یا صفر) f(n)=n (این تابع هیچگاه مقدار یک را نمی گیرد.)

ali_hp
13-12-2006, 00:33
با سلام

فرض کنید:


[ برای مشاهده لینک ، لطفا با نام کاربری خود وارد شوید یا ثبت نام کنید ]

ثابت کنید:


[ برای مشاهده لینک ، لطفا با نام کاربری خود وارد شوید یا ثبت نام کنید ]

موفق باشید.

ارسال متن: شنبه 18 آذر 1385
سلام
[ برای مشاهده لینک ، لطفا با نام کاربری خود وارد شوید یا ثبت نام کنید ]
با استفاده از نامساوی کوشی داریم:
[ برای مشاهده لینک ، لطفا با نام کاربری خود وارد شوید یا ثبت نام کنید ]
در گام اخر هم از نامساوی میانگین حسابی هندسی استفاده شده است.
اقای مفیدی من فکر می کنم اگر مسایل هفته ساده تر باشند بهتر است و افراد بیشتری در بحث شرکت می کنند واز بحث استفاده می کنند.بیشتر مسایل هفته ازمسایل المپیادها انتخاب می شوند که حل این مسایل نیازمندصرف وقت وحوصله نسبتا زیادی است.

mofidy1
13-12-2006, 10:31
سلام

اقای مفیدی من فکر می کنم اگر مسایل هفته ساده تر باشند بهتر است و افراد بیشتری در بحث شرکت می کنند واز بحث استفاده می کنند.بیشتر مسایل هفته ازمسایل المپیادها انتخاب می شوند که حل این مسایل نیازمندصرف وقت وحوصله نسبتا زیادی است.

با سلام خدمت دوست عزیزم آقای حسین پوران

علی آقا، با این مطلب که اکثر مسائل هفته از المپیادهای خارجی و داخلی انتخاب می شوند، با شما موافقم، اما با این نظر شما که حل این مسایل نیازمندصرف وقت وحوصله نسبتا زیادی است، خیلی موافق نیستم، زیرا بین این مسائل، مساله های ساده ای نیز وجود دارند که با کمی صرف وقت حل می شوند؛ هر چند که قبول دارم مسائل بسیار مشکل هم در بین اینها وجود دارد. متاسفانه معیار دقیقی برای سختی و آسانی یک مساله ریاضی وجود ندارد و این مطلب تا حدی سلیقه ای است. فکر می کنم دلیل اینکه بخش مسائل هفته خیلی مورد توجه قرار نگرفته، فقط سختی سوالات نیست. احتمالا مطلع نبودن دانش آموزان و دانشجویان از موجودیت این بخش، آشنا نبودن آنها با طریقه نوشتن نمادهای ریاضی در صفحات وب و فرصت کم آنها به علت اشتغال به تحصیل، چند علت مهم دیگر است. اولی را می توان با تبلیغات بیشتر در اینترنت و نیز در دبیرستانها و دانشگاهها و نیز در بین دوستان و آشنایان تا حدی رفع کرد. برای رفع مشکل دومی نیز به فکر نوشتن مقاله ای مفصل در این زمینه هستم و سومی نیز به مرور زمان حل می شود، البته اگر سیستم آموزشی نامتعادل و غیر استاندارد ما اجازه دهد!! فکر می کنم با پیدا کردن افراد علاقمند به ریاضی و دادن مسئولیت به آنها این مشکل نیز حل شود. باید در آینده به فکر کارهای تشویقی برای جذب افراد باشیم.

با این حال با توجه به این سخن زیبای امیر مومنان (ع) که «فی تقلب الاحوال علم جواهر الرجال» و به خاطر ارزش بسیاری که برای نظرات شما قائلم، از این هفته تا آخر زمستان، سطح سوالات را کاهش می دهم و اگر احساس کردم مساله مشکل است، راهنمایی خواهم کرد.

از شما و آقای منبتی هم خواهش می کنم که به سوالات بلافاصله پاسخ ندهید و راه حلهایتان را در روزها پنجشنبه و جمعه در اتاق قرار دهید. بنده نیز کماکان در روز جمعه، اگر روش دیگری مد نظر داشتم مطرح خواهم کرد.

از بذل توجه شما بسیار ممنونم.

موفق باشید.

m_honarmand_j
13-12-2006, 16:50
سلام دوستان
بالاخره تاپیکی با عنوان اتاق تر کیبیات را شروع کردم . از این به بعد یؤالات ترکیبیات رو در اونجا مطرح می کنم . اگر علاقه مندید حتما سر بزنید . این تاپیک در همین سایت p30world در بخش ریاضیات است . امیدوارم که خوشتون بیاد .
فعلا .

m_honarmand_j
14-12-2006, 16:09
سلام بر دوستداران ریاضی
من کار خود را در اتاق ترکیبیات شروع کردم و تا اینجا دو سؤال نیز مطرح کردم . تمام سعی من بر این است که در همه ی سطوح ترکیبیات مسئله مطرح کنم . اگه وقت کردید به ما هم سر بزنید .

m_honarmand_j
14-12-2006, 16:20
سلام دوستان
در مورد سؤالی که در باره ی دنباله ها مطرح کردم (دنباله ای به طول n شامل m تا 01 ) بهتره که حالات بندی کنید تا راحت تر حل بشه . ببینید اگر این دنباله با 0 شروع شود و با 0 یا 1 پایان یابد یا اینکه با 1 شروع و با 0 یا 1 تمام شود چه می شود. می توانید فرض کنید که تمام دنباله از 0 تشکیل شده و می خواهیم تهدادی از آنها را تبدیل به 1کنیم .
باز هم تکرار می کنم که من از این به بعد سؤالات ترکیبیات را در اتاق ترکیبیات مطرح می کنم .
با تشکر از همه ی دوستان

m_honarmand_j
14-12-2006, 16:28
سلام بروبچ
در يك n ضلعي محدب تمام قطر هاي آن را رسم مي كنيم . اگر هيچ سه قطري همراس نباشند , n ضلعي به چند بخش تقسيم مي شود ؟

سلام بر دوستان . من برای حل این مسئله دوتا راح حل پیشنهاد می کنم . یک از طریق مجموع زوایا استفاده کنید و ببینید هر تقاطع ایجاد شده چه مقدار یه مجموع زوایا اضافه می کند و ...
راه دیگر استفاده از فرمول اویلر در مورد گراف ها است .
امید وارم که تونسته باشم راهنمایی خوبی کرده باشم .

m_honarmand_j
14-12-2006, 16:33
سلام
در روزگاران قديم , از طرف فرمانرواي شهري اين اطلاعيه صادر شده است : ((مسلم شده است در شهر زناني وجود دارند كه به شوهران خود خيانت ميكنند, و مسلم شده است كه بر خيانت هر يك از اين زنان , همه ي مردم شهر به غير از شوهر ايشان اطلاع دارند . از مردان خواسته مي شود كه فقط از روي تعقل و بدون تفحص از ديگران درباره ي زنان خود بينديشند و هر مرد كه در يافت زنش به وي خيانت مي كند بامداد روز بعد آن زن را به قتل برساند .)) ثابت كنيد بلاخره روزي فرا ميرسد كه تمام زنان خيانت كار كشته شده باشند .

سلام
اگر تعداد زنان خیانت کار را برابر m بگیریم با استقرا بر روی m می توان ثابت کرد که تمام زنان خیانتکار در روز m ام کشته می شوند . ادامه ی اثبات بر عهده ی خواننده است :tongue:

m_honarmand_j
14-12-2006, 16:40
سلام .
تعدادي خانه در دور يك ميدان قرار دارند به طوري كه سقف هر خانه يا قرمز است يا سبز و تعداد همسايه هاي هر خانه عددي فرد است . از يك خانه شروع كرده و رنگ آن را به رنگي كه بيشتر همسايه هايش دارند در مي آوريم . اين كار را دور تا دور ميدان انجام مي دهيم . ثابت كنيد به مرحله اي مي رسيم كه ديگر رنگ خانه ها عوض نمي شود .

سلام
این مسئله را به گراف تبدیل می کنیم . برای هر خانه یک راس و خانه های همسایه را با یالی به هم متصل می کنیم . در این مسئله یالهایی که دوسر آنها دارای رنگ های متفاوت است را در نظر می گیریم . مشاهده می شود که در هر مرحله تعداد آنها کم می شود و حد اقل تعداد این یالها 0 است . پس این الگوریتمی که انجام می دهیم پایان پذیر است . :blink: :laughing:

m_honarmand_j
14-12-2006, 16:49
سلام
n نقطه آبي و n نقطه قرمز در صفحه وجود دارد . مي خواهيم هر نقطه آبي را به يك نقطه قرمز با خط هاي صاف وصل كنيم به طوري كه اين خطوط همديگر را قطع نكنند . ثابت كنيد اين كار همواره امكان پذير است . :tongue:

سلام
هر دوخطی ک با هم تقاطع دارند را با هم عوض می کنیم . مثلا اگر قرمز1 به آبی1 و قرمز2 به آبی2 متصل هست و این دو خط با هم تقاطع دارند آنهارا به قرمز1_آبی2 و قرمز2_آبی1 تغییر میدهیم .
اثبات اینکه این کار پایان پذیر است بر عهده ی خودتان است . :rolleye:

m_honarmand_j
14-12-2006, 16:54
سلام .
يك جمعي از نمايندگان كشور ها مي خواهند در دورديف صندلي روبهروي هم بشينند .(دوطرف ميز) . هر كس حد اكثر 3 دشمن دارد . ثابت كنيد مي توان اين افراد را در اين دو رديف صندلي نشاند به طوري كه هر كس حد اكثر يك دشمن در رديفي كه نشسته است داشته باشد .

سلام
همه را به طور دلخواه می چینیم . سپس هر کس که ناراضی است(بیش از یک دشمن در طرف خودش دارد) را به طرف مقابل منتقل می کنیم . در هر مرحله تعداد افراد ناراضی کم میشود و حداقل این تعداد 0 است پس این کار پایان می یابد . :laughing: :laughing: :laughing:

m_honarmand_j
14-12-2006, 17:39
سلام بر دوستان
تر کوندم . جواب چند تا از سؤالات و یا راهنمایی آنها رو گذاشتم . اگه مشکلی بود مطرح کنید .
7 تا سؤال ناب تو اتاق تر کیبیات گذاشتم . دیگه چی میی خواهید .
اگه تونستید به اتاق تر کیبیات سر بزنید .

رسولی حسین
17-12-2006, 02:55
دوستان سلام خواهش میکنم اگر امکان داره جواب سوال منو زودتر بدین
دنباله زیر همگراست یا واگرا چرا اگر همگرا است به چه عددی






[ n^7/3^n - 1/(2n-1)^2 ] ∑

eh_mn
17-12-2006, 19:13
دوستان سلام خواهش میکنم اگر امکان داره جواب سوال منو زودتر بدین
دنباله زیر همگراست یا واگرا چرا اگر همگرا است به چه عددی






[ n^7/3^n - 1/(2n-1)^2 ] ∑

با سلام.
آيا منظور شما سري
[ برای مشاهده لینک ، لطفا با نام کاربری خود وارد شوید یا ثبت نام کنید ]
است؟

ali_hp
17-12-2006, 19:56
دوستان سلام خواهش میکنم اگر امکان داره جواب سوال منو زودتر بدین
دنباله زیر همگراست یا واگرا چرا اگر همگرا است به چه عددی






[ n^7/3^n - 1/(2n-1)^2 ] ∑

سلام
همگراست اول با روشهای مقدماتی ثابت می کنم که دنباله همگراست و بعد با استفاده از روشی که خودم هم دقیقا نمی دانم چیست مقدار حد را محاسبه می کنم!
اثبات همگرایی:
قراردهید
[ برای مشاهده لینک ، لطفا با نام کاربری خود وارد شوید یا ثبت نام کنید ]
ثابت می کنیم دنباله های A_n, B_n همگرایند و نتیجه می شود S_n نیز همگراست.
َ A_n وB_n صعودی هستند پس کافی است ثابت کنیم کران بالا دارند.
برای k>1 به سادگی می توان ثابت کرد که:
[ برای مشاهده لینک ، لطفا با نام کاربری خود وارد شوید یا ثبت نام کنید ]
پس داریم:
[ برای مشاهده لینک ، لطفا با نام کاربری خود وارد شوید یا ثبت نام کنید ]
پس دنباله B_n صعودی و کراندار است بنابراین همگرااست.
برای اثبات کراندار بودن A_n ابتدا لم زیر را ثابت می کنیم:
لم.برای هر x>1 و k طبیعی عدد طبیعی مانند N موجود است به طوریکه برای هر n>N داشته باشیم:
[ برای مشاهده لینک ، لطفا با نام کاربری خود وارد شوید یا ثبت نام کنید ]
اثبات لم:فرض کنید x=1+t که t>0برای n>k طبق بسط دو جمله ای داریم:
[ برای مشاهده لینک ، لطفا با نام کاربری خود وارد شوید یا ثبت نام کنید ]
از طرفی:
[ برای مشاهده لینک ، لطفا با نام کاربری خود وارد شوید یا ثبت نام کنید ]
پس p(n)l چند جمله ای از درجه k+1 بر حسب n است.و ضریب جمله باتوان k+1 ان مثبت است.پس m ای وجود دارد که برای n>m داشته باشیم p(n)>n^k .پس برای n>max{k+1,m}l داریم:
[ برای مشاهده لینک ، لطفا با نام کاربری خود وارد شوید یا ثبت نام کنید ]
حال اگر در لم قرار دهیم x=3,k=9 عدد طبیعی N وجود دارد که برای n>N داریم :
[ برای مشاهده لینک ، لطفا با نام کاربری خود وارد شوید یا ثبت نام کنید ]
پس برای n>N داریم:
[ برای مشاهده لینک ، لطفا با نام کاربری خود وارد شوید یا ثبت نام کنید ]
پس A_n نیز کران بالا دارد و با توجه به صعودی بودنA_n نتیجه می شود همگراست.
برای محاسبه حد A_n دقت کنید که به ازای x های بین صفر ویک داریم:
[ برای مشاهده لینک ، لطفا با نام کاربری خود وارد شوید یا ثبت نام کنید ]
با ادامه دادن روش بالا که بر پایه رابطه زیر است:
[ برای مشاهده لینک ، لطفا با نام کاربری خود وارد شوید یا ثبت نام کنید ]می توان مقدار :
[ برای مشاهده لینک ، لطفا با نام کاربری خود وارد شوید یا ثبت نام کنید ]
را حساب کرد و با قراردادن x=1/3 حد A_n بدست می اید.
حد B_n
می توان ثابت کرد که:
[ برای مشاهده لینک ، لطفا با نام کاربری خود وارد شوید یا ثبت نام کنید ]
همچنین :
[ برای مشاهده لینک ، لطفا با نام کاربری خود وارد شوید یا ثبت نام کنید ]
در نتیجه داریم:
[ برای مشاهده لینک ، لطفا با نام کاربری خود وارد شوید یا ثبت نام کنید ]
حد S_n هم برابر حدA_n منهای حد B_n است.

ali_hp
18-12-2006, 00:42
سلام
n نقطه آبي و n نقطه قرمز در صفحه وجود دارد . مي خواهيم هر نقطه آبي را به يك نقطه قرمز با خط هاي صاف وصل كنيم به طوري كه اين خطوط همديگر را قطع نكنند . ثابت كنيد اين كار همواره امكان پذير است . :tongue:
سلام
مثال نقض:روی یک خط یک نقطه دلخواه مثل A در نظر می گیریم n نقطه ابی را در سمت راستA وروی خط و n نقطه قرمز را در سمت چپ A وروی خط در نظر می گیریم.

mofidy1
18-12-2006, 08:00
با سلام

فرض کنید:


[ برای مشاهده لینک ، لطفا با نام کاربری خود وارد شوید یا ثبت نام کنید ]

ثابت کنید:


[ برای مشاهده لینک ، لطفا با نام کاربری خود وارد شوید یا ثبت نام کنید ]

موفق باشید.

ارسال متن: شنبه 18 آذر 1385

با سلام

از آقای حسین پوران به خاطر حل مساله تشکر می کنم. برای دیدن راه حل ایشان به لینک زیر مراجعه فرمایید:

[ برای مشاهده لینک ، لطفا با نام کاربری خود وارد شوید یا ثبت نام کنید ]

موفق باشید.

ارسال متن: دوشنبه 27 آذر 1385

mofidy1
18-12-2006, 08:48
با سلام

فرض کنید:


[ برای مشاهده لینک ، لطفا با نام کاربری خود وارد شوید یا ثبت نام کنید ]

با استفاده از نامساوی زیر


[ برای مشاهده لینک ، لطفا با نام کاربری خود وارد شوید یا ثبت نام کنید ]

و به وسیله استقراء ثابت کنید:


[ برای مشاهده لینک ، لطفا با نام کاربری خود وارد شوید یا ثبت نام کنید ]

موفق باشید.

ارسال متن: دوشنبه 27 آذر 1385

mbsh
19-12-2006, 21:45
خواهش مي كنم يكي همين امشب كمكم كنه.
در مورد الگوريتم پريم مختصري توضيحات مي خوام.

Vmusic
19-12-2006, 22:33
شلام بچه ها قفلید
یه جا رد خواست بده
ممنون

mofidy1
20-12-2006, 00:10
خواهش مي كنم يكي همين امشب كمكم كنه.
در مورد الگوريتم پريم مختصري توضيحات مي خوام.

با سلام

این آلگوریتم برای پیدا کردن یک درخت فراگیر با وزن می نیمم است. ابتدا یک راس و یک ضلع متصل به آن را با وزن می نیمم انتخاب کنید. در مراحل بعد یک ضلع را به وزن می نیمم طوری انتخاب کنید که یک راس انتخاب نشده را به راس انتخاب شده قبلی متصل کند. این فرایند را تا انتخاب تمامی رئوس ادامه دهید.

موفق باشید.

m_honarmand_j
20-12-2006, 16:45
سلام علی جان
مثال نقضت درسته ولی تو مسئله ی اصلی اومده که هیچ سه نقطه ای بر روی یک خط راست واقع نمی باشند و من چون سوال و توند نوشتم فراموش کردم این نکته رو بگم . شما به بزرگیتون ببخشید .

mbsh
22-12-2006, 11:30
خيلي ممنون در مورد وزن مينيمم هم مي شه كمي توضيح بديد.
اگه يه كتاب فارسي خوب هم در مورد اين الگوريتم معرفي كنيد ممنون مي شم.

mbsh
22-12-2006, 11:35
من مي خواستم اون مطلبو اين جا بنويسم نمي دونم چرا تو يه صفحه ي ديگه وارد شد؟
از mofidy1 هم تشكر مي كنم. اگه لطف كنيد در مورد وزن مينيمم هم يه ذره توضيح بديد ممنون مي شم.
اگه يه كتاب فارسي هم براي اين الگوريتم معرفي كنيد ديگه خيلي خيلي ممنون

mofidy1
22-12-2006, 12:23
خيلي ممنون در مورد وزن مينيمم هم مي شه كمي توضيح بديد.
اگه يه كتاب فارسي خوب هم در مورد اين الگوريتم معرفي كنيد ممنون مي شم.

یک گراف را وزندار گوییم اگر هر ضلع آن برچسب عددی داشته باشد که آنرا وزن آن ضلع گوییم. اگر درخت فراگیری از یک گراف به گونه ای پیدا کنیم که مجموع وزن اضلاع، کمترین باشد، گوییم درختی با وزن می نیمم یافته ایم.

کتاب: نخستین درس در ریاضیات گسسته، تالیف ایان اندرسن، ترجمه مرتضی اسماعیلی، مرکز نشر دانشگاه صنعتی اصفهان، صفحه 57

موفق باشید.

mehrtooka
22-12-2006, 12:30
تفاوت مشتق با حد چیست؟

mehrtooka
22-12-2006, 12:38
لطفاَ جواب بدید

mehrtooka
22-12-2006, 12:43
لطفاَ جواب این سوال را هر چه زودتر به اطلاع اینجانب برسانید.با تشکر

mofidy1
22-12-2006, 13:13
تفاوت مشتق با حد چیست؟

با سلام

مشتق، یکی از مباحث مربوط به حد است. برای محاسبه مشتق یک تابع ، باید حد کسر خاصی را که وابسته به آن تابع است، محاسبه کنیم.

موفق باشید.

mofidy1
22-12-2006, 17:09
با سلام

فرض کنید:


[ برای مشاهده لینک ، لطفا با نام کاربری خود وارد شوید یا ثبت نام کنید ]

با استفاده از نامساوی زیر


[ برای مشاهده لینک ، لطفا با نام کاربری خود وارد شوید یا ثبت نام کنید ]

و به وسیله استقراء ثابت کنید:


[ برای مشاهده لینک ، لطفا با نام کاربری خود وارد شوید یا ثبت نام کنید ]

موفق باشید.

ارسال متن: دوشنبه 27 آذر 1385

با سلام

اگر n=1 مطلب بدیهی است. حال با استفاده از فرض استقراء و نیز نامساوی ذکر شده می توان نوشت:


[ برای مشاهده لینک ، لطفا با نام کاربری خود وارد شوید یا ثبت نام کنید ]

که حکم استقراء را ثابت می کند.

موفق باشید.

ارسال متن: جمعه 1 دی 1385

mehrtooka
22-12-2006, 18:28
پس در واقع تفاوتی بین مشتق و حد وجود ندارد.

mofidy1
22-12-2006, 18:32
نه اینطور نیست. مبحث حد، بسیار گسترده تر از مشتق است و مشتق یکی از موضوعات مربوط به آن است.

mehrtooka
22-12-2006, 18:45
یکی از بچه ها در دانشگاه از من این سوال یعنی تفاوت بین حد و مشتق را پرسید به همین دلیل خیلی دوست دارم بیشتر بدانم.با تشکر از شما.

mofidy1
22-12-2006, 19:12
یکی از بچه ها در دانشگاه از من این سوال یعنی تفاوت بین حد و مشتق را پرسید به همین دلیل خیلی دوست دارم بیشتر بدانم.با تشکر از شما.

کاربر گرامی، در دانشگاه از هر استاد ریاضی که مایل هستید، این سوال را بپرسید. این مطلب را ایشان در عرض چند دقیقه برایتان مفصلا توضیح خواهند داد.

موفق باشید.

mbsh
22-12-2006, 23:32
سلام و تشكر
امروز نمي تونم كتابو گير بيارم و فعلا دارم از كتاب قليزاده از موسسه انتشارات علمي مطالعه مي كنم.
خواهش مي كنم اين دو تا قضيه رو هم خيلي ساده برام اثبات كنيد.
اثبات كنيد كه درخت حاصل از الگوريتم هاي كروسكال و پريم درخت هاي پوشاي مينيمم هستند.
خيلي ممنونم.

m_honarmand_j
23-12-2006, 19:58
سلام بر دوستان
فرا رسیدن ایام امتحانات رو به بعضی ها تبریک و به بعضی ها تصلیت عرض می کنم. :tongue:

m_honarmand_j
23-12-2006, 20:05
سلام
در ضرب رو عدد سه رقمی حداکثر چند 9 میتواند وجود داشته باشد؟

mofidy1
23-12-2006, 23:46
با سلام

می دانیم که اگر مشتق یک تابع در یک بازه صفر باشد، آن تابع در آن بازه، مقدار ثابت دارد. با توجه به این نکته ، دو مساله زیر را حل کنید:

الف) تابع f در شرط زیر صدق می کند. ثابت کنید f تابعی ثابت است:


[ برای مشاهده لینک ، لطفا با نام کاربری خود وارد شوید یا ثبت نام کنید ]

ب) ثابت کنید تابع زیر ، تابعی ثابت است:


[ برای مشاهده لینک ، لطفا با نام کاربری خود وارد شوید یا ثبت نام کنید ]

موفق باشید.

ارسال متن: شنبه 2 دیماه 1385

ali1234
24-12-2006, 00:01
سلام بر دوستان
فرا رسیدن ایام امتحانات رو به بعضی ها تبریک و به بعضی ها تصلیت عرض می کنم. :tongue:

سلام دوست عزيز
"تسليت" نه "تصليت":tongue:

mbsh
24-12-2006, 00:08
چرا هيچ كي جوابمو نمي ده؟
من اثبات قسه هايي رو كه مي گن الگوريتماي كروسكال و پريم توليد درخت پوشاي مينيمم مي كنن رو مي خوام. آقا بايد برا يه عده توضيح بدم. كمكم كنيد.

ali1234
24-12-2006, 15:13
با سلام

می دانیم که اگر مشتق یک تابع در یک بازه صفر باشد، آن تابع در آن بازه، مقدار ثابت دارد. با توجه به این نکته ، دو مساله زیر را حل کنید:

الف) تابع f در شرط زیر صدق می کند. ثابت کنید f تابعی ثابت است:


[ برای مشاهده لینک ، لطفا با نام کاربری خود وارد شوید یا ثبت نام کنید ]

ب) ثابت کنید تابع زیر ، تابعی ثابت است:


[ برای مشاهده لینک ، لطفا با نام کاربری خود وارد شوید یا ثبت نام کنید ]

موفق باشید.

ارسال متن: شنبه 2 دیماه 1385
[ برای مشاهده لینک ، لطفا با نام کاربری خود وارد شوید یا ثبت نام کنید ]

mir@
24-12-2006, 16:28
سلام
[ برای مشاهده لینک ، لطفا با نام کاربری خود وارد شوید یا ثبت نام کنید ]

عزیزان من یک عمره به دنبال معکوس تابع بالا می گردم. کسی می تونه کمک کنه؟؟
با سپاس

رسولی حسین
24-12-2006, 16:55
با سلام خدمت دوستان
از دوست خوبم ali_ph به خاطر حل سوال دنباله تشکر میکنم البته خیلی دیر امدم چون سرم شلوغ بود وقت نکردم

ali_hp
24-12-2006, 19:57
سلام
[ برای مشاهده لینک ، لطفا با نام کاربری خود وارد شوید یا ثبت نام کنید ]

عزیزان من یک عمره به دنبال معکوس تابع بالا می گردم. کسی می تونه کمک کنه؟؟
با سپاس
سلام
طبق فرمول کاردانو یکی از ریشه های معادله
[ برای مشاهده لینک ، لطفا با نام کاربری خود وارد شوید یا ثبت نام کنید ]
عبارت است از:
[ برای مشاهده لینک ، لطفا با نام کاربری خود وارد شوید یا ثبت نام کنید ]
حال اگر قرار دهیم p=1و q=-y می توانیم ریشه یکتای معادله
[ برای مشاهده لینک ، لطفا با نام کاربری خود وارد شوید یا ثبت نام کنید ]
را بر حسب y بدست اوریم و از انجا معکوس تابع بدست می اید:
[ برای مشاهده لینک ، لطفا با نام کاربری خود وارد شوید یا ثبت نام کنید ]
در مورد تابع مثلثاتی هم اگه منظورت مساله هفته است که باید منهای صورت اولین کسرو باضافه کنی.در غیر اینصورت هم تابعی که معرفی کردی تابع ثابت نیست(صورت کسر و اشتباه به حاصلضرب تبدیل کردی)

eh_mn
24-12-2006, 21:13
چرا هيچ كي جوابمو نمي ده؟
من اثبات قسه هايي رو كه مي گن الگوريتماي كروسكال و پريم توليد درخت پوشاي مينيمم مي كنن رو مي خوام. آقا بايد برا يه عده توضيح بدم. كمكم كنيد.
با سلام
اثبات درستي الگوريتم Prim با استفاده از قضيه زير انجام مي شود.
قضيه.
فرض كنيد G گرافي با مجموعه رئوس V و مجموعه يالهاي E يك گراف بدون جهت وزندار باشد. فرض كنيد N يرمجموعه اي سره از V است. همچنين فرض كنيد T زيرمجموعه اي از يالهاي G باشد كه مي تواند به درخت پوشاي مينيمم منتهي گردد و هيچ يالي از T با هيچ گرهي از N تماس نداشته باشد ، چنانچه e كوتاهترين يالي باشد كه از گرهي خارج از N با يكي از گره هاي N تماس دارد ، آنگاه T+e (يعني مجموعه T به همراه يال e) نيز مي تواند به درخت پوشاي مينيمم منتهي گردد.

برهان.
طبق فرض T بگونه ايست كه مي تواند به يك درخت پوشاي مينيمم منتهي شود. نام اين درخت پوشاي مينيمم را U مي گذاريم. اگر يال e در U باشد كه چيزي براي اثبات نمي ماند. اگر يال e در U نباشد آن را به U مي افزاييم. واضح است كه يك دور ايجاد خواهد كرد. در اين دور حداقل يك راس از N وجود دارد. (همان راسي كه e با آن تماس دارد)
پس در دور حداقل يك يال ديگر كه با راسي از N تماس دارد وجود دارد. اين لبه را 'e مي ناميم. چون ارزش 'e حداقل به اندازه e است با حذف 'e دور از بين رفته و درخت حاصل يك درخت پوشاي مينيمم است. واضح است كه 'e در T نيست (زيرا بنا به فرض هيچ يالي از T با هيچ گرهي از N تماس ندارد) بنا براين T زيرمجموعه درخت جديد نيز مي باشد و لذا
T+e در درخت پوشاي مينيمم جديد خواهد بود.

m_honarmand_j
24-12-2006, 23:51
سلام بر دوستان
مدتی هست که من در مورد ترکیبیات دنبال منبع سوال برای حل کردن می گردم . لطفا اگه کسی منبع مناسبی مثل کتاب ، سایت یا هر منبع دیگه ای سراغ داره بهم معرفی کنه .
ممنون می شم .

ali_hp
25-12-2006, 02:28
سلام
فکر نمی کنم سوال ساده ای باشه ولی به نظر من خیلی جالبه:
فرض کنید P چند جمله ای با ضرایب صحیح باشدوضریب پیشرو ان یک باشد وd عددی طبیعی به طوریکه درجه P برd بخش پذیر باشد و معادله
[ برای مشاهده لینک ، لطفا با نام کاربری خود وارد شوید یا ثبت نام کنید ]
نا متناهی جواب برای m,n طبیعی داشته باشد.ثابت کنید چند جمله ای Q با ضرایب صحیح وجود دارد به طوری که برای هر x:
[ برای مشاهده لینک ، لطفا با نام کاربری خود وارد شوید یا ثبت نام کنید ]

mir@
25-12-2006, 14:23
سلام
طبق فرمول کاردانو یکی از ریشه های معادله
[ برای مشاهده لینک ، لطفا با نام کاربری خود وارد شوید یا ثبت نام کنید ]
عبارت است از:
[ برای مشاهده لینک ، لطفا با نام کاربری خود وارد شوید یا ثبت نام کنید ]
پس داریم:
[ برای مشاهده لینک ، لطفا با نام کاربری خود وارد شوید یا ثبت نام کنید ]
در مورد تابعی مثلثاتی هم اگه منظورت مساله هفته است که باید منهای صورت اولین کسرو باضافه کنی.در غیر اینصورت هم تابعی که معرفی کردی تابع ثابت نیست(صورت کسر و اشتباه به حاصلضرب تبدیل کردی)

سلام علی آقا
با تشکر از زحمتی که کشیدید
من احساس می کنم پاسخ شما خیلی درست نیست. یا لااقل یک اشکال کوچک داره چون
f(1)=2 اما اگر g را معکوس f فرض کنیم یعنی همون رابطه ای که شما دادی (بالا) g(2)=-1
چرا؟

ali_hp
25-12-2006, 14:32
سلام علی آقا
با تشکر از زحمتی که کشیدید
من احساس می کنم پاسخ شما خیلی درست نیست. یا لااقل یک اشکال کوچک داره چون
f(1)=2 اما اگر g را معکوس f فرض کنیم یعنی همون رابطه ای که شما دادی (بالا) g(2)=-1
چرا؟

سلام
حق با شماست. توی همون پست قبلی(پست 553) جوابو ویرایش می کنم.

mir@
25-12-2006, 15:06
آهان فهمیدم
شما به y=q قرار دادید که باید می گذاشتید y=-q


[ برای مشاهده لینک ، لطفا با نام کاربری خود وارد شوید یا ثبت نام کنید ]

ما جمعه امتحان ریاضی برای ورودی دکترای شریف داشتیم. اون سوال حد هم توش بود. متاسفانه نمی دونم چرا سر جلسه راه حلش به ذهنم نرسید (خاک بر فرق سرم:puke: ). حالا این درسته؟
البته درست یادم نیست همین بود یا نه. ولی در مجموع به نظر من امتحان سختی بید. شما عزیزان هیچ کدوم شرکت نکردید؟

ali_hp
25-12-2006, 15:38
بله اشتباهم همون بوده.
راه حلتم درسته یه سوال شبیه این سوال قبلا تو همین اتاق مطرح شده بودو از سه راه هم حل شده بود که یکیش تقریبا همین راه حل شما بود
این سوال:[ برای مشاهده لینک ، لطفا با نام کاربری خود وارد شوید یا ثبت نام کنید ]
اینم سه تا راه حلش:
[ برای مشاهده لینک ، لطفا با نام کاربری خود وارد شوید یا ثبت نام کنید ]
[ برای مشاهده لینک ، لطفا با نام کاربری خود وارد شوید یا ثبت نام کنید ]
[ برای مشاهده لینک ، لطفا با نام کاربری خود وارد شوید یا ثبت نام کنید ]

mbsh
25-12-2006, 18:23
با تشكر از eh-mn
اين براي هر دو تا الگوريتم ثابت مي كنه كه درخت هاي توليد شده min هستن؟

eh_mn
26-12-2006, 10:44
با تشكر از eh-mn
اين براي هر دو تا الگوريتم ثابت مي كنه كه درخت هاي توليد شده min هستن؟
سلام
برای اثبات درستی هر دو الگوریتم می توان از این قضیه استفاده کرد. (به نقل از کتاب طراحی الگوریتم ها نوشته دکتر نقیب زاده)

mir@
26-12-2006, 12:04
سلام
جواب تحلیلی این مسئله رو می خواستم.
[ برای مشاهده لینک ، لطفا با نام کاربری خود وارد شوید یا ثبت نام کنید ]

mehdi0016
26-12-2006, 15:12
سلام
یه فرمول که با اون بشه فهمید که آیا نقطه مثلا A به مختصات (x1,y1) روی محیط دایره ای به شعاع r قرار دارد یا نه ؟
ممنون

ali_hp
26-12-2006, 15:14
سلام
جواب تحلیلی این مسئله رو می خواستم.
[ برای مشاهده لینک ، لطفا با نام کاربری خود وارد شوید یا ثبت نام کنید ]
سلام. جواب تحلیلی یعنی چی؟

AbbasGholami
26-12-2006, 15:44
سلام
یه فرمول که با اون بشه فهمید که آیا نقطه مثلا A به مختصات (x1,y1) روی محیط دایره ای به شعاع r قرار دارد یا نه ؟
ممنون

سلام
مرکز دایره رو مشخص نکردی. حالا فرض میکنیم مرکز دایره (a,b) باشد. حالا معادله یک دایره با این مختصات و شعاع r چیه؟
x)(x-a)^2+(y-b)^2=r^2
حالا نقطه (x1,y1) رو داخل این معادله بگذار (یعنی به جای x,y این نقطه رو بگذار x1,y1)، اگر توی معادله صدق کرد یعنی این نقطه روی محیط دایره قرار گرفته.

AbbasGholami
26-12-2006, 16:11
به جای x از a^2 و به جای y از b^2 استفاده کنید. بعد از آن جمله پایینی را از بالایی کم کنید به صورت زیر در می آید:
a-b-(a^2-b^2)=a-b-(a-b)(a+b)=(a-b)*[1-(a+b)]=4
حالا به جای 4 از حالتهای مختلف ضرب دو عدد استفاده کنید:
4=2*2
4=2-*2-
4=1*4
4=4*1
4=1-*4-
4=4-*1-
و بعد نظیر به نظیر مساوی جملات مجهول قرار میدهیم. میبینیم که جواب میشود:
a=2 , b=3 یعنی:
x=4 , y=9

mir@
27-12-2006, 19:09
سلام. جواب تحلیلی یعنی چی؟

سلام
منظورم اینه که جوابش مشخصه راه حلش رو می خواستم.


به جای x از a^2 و به جای y از b^2 استفاده کنید. بعد از آن جمله پایینی را از بالایی کم کنید به صورت زیر در می آید:
a-b-(a^2-b^2)=a-b-(a-b)(a+b)=(a-b)*[1-(a+b)]=4
حالا به جای 4 از حالتهای مختلف ضرب دو عدد استفاده کنید:
4=2*2
4=2-*2-
4=1*4
4=4*1
4=1-*4-
4=4-*1-
و بعد نظیر به نظیر مساوی جملات مجهول قرار میدهیم. میبینیم که جواب میشود:
a=2 , b=3 یعنی:
x=4 , y=9
آقای غلامی معلوم نیست که جواب ها حتماً اعداد صحیح باشه.
راه حل دقیق تر و بهتری بدون امتحان کردن وجود نداره؟

m_honarmand_j
28-12-2006, 18:08
سلام بر دوستان
فکر کنم از هفته ی بعد امتحانات دانشجویان هم شروع می شه . امید وارم همه موفق باشید . به اتاق تر کیبیات هم سر بزنید . خوشحال می شم.
دوستان ، آیا کسی کتاب فارسی خوب در مورد گراف سراغ داره ؟ (به جز باندی مورتی و وست . اگه ترجمه ی دیستل رو کسی داشته باشه محشر میشه چون من اینگلیسی شو دارم)

ali_hp
29-12-2006, 03:05
سلام
منظورم اینه که جوابش مشخصه راه حلش رو می خواستم.

سلام
امیرجان میشه بگی جوابش چیه؟
من ثابت کردم دستگاه معادلات سه تا جواب دیگه (به جز جوابx=4,y=9) داره ولی نتونستم جوابشو بدست بیارم.
اگر x=a^2و y=b^2 داریم:
[ برای مشاهده لینک ، لطفا با نام کاربری خود وارد شوید یا ثبت نام کنید ]
اگرa=2 جواب x=4وy=9 بدست می اید. می توان ثابت کردعبارت درجه سوم هم سه ریشه حقیقی دارد.که کن نتوانستم ریشه های این معادله درجه سه را به صورت مرسوم بیان کنم.

b-z
29-12-2006, 11:42
[ برای مشاهده لینک ، لطفا با نام کاربری خود وارد شوید یا ثبت نام کنید ]

در تقسيم بدون باقيمانده فوق (-) ها نماينده ارقامي هستند كه ممكن است متمايزنباشند . مقسوم و مقسوم عليه و خارج قسمت را تعيين كنيد .
به نظر ساده مياد كسي ميتونه كمك كنه ؟!

mofidy1
29-12-2006, 14:05
با سلام

می دانیم که اگر مشتق یک تابع در یک بازه صفر باشد، آن تابع در آن بازه، مقدار ثابت دارد. با توجه به این نکته ، دو مساله زیر را حل کنید:

الف) تابع f در شرط زیر صدق می کند. ثابت کنید f تابعی ثابت است:


[ برای مشاهده لینک ، لطفا با نام کاربری خود وارد شوید یا ثبت نام کنید ]

ب) ثابت کنید تابع زیر ، تابعی ثابت است:


[ برای مشاهده لینک ، لطفا با نام کاربری خود وارد شوید یا ثبت نام کنید ]

موفق باشید.

ارسال متن: شنبه 2 دیماه 1385

با سلام

از ali1234 که در پست 550 مساله این هفته را حل کردند، تشکر می کنم. برای دیدن راه حل ایشان به لینک زیر مراجعه فرمایید:

[ برای مشاهده لینک ، لطفا با نام کاربری خود وارد شوید یا ثبت نام کنید ]

موفق باشید.

ارسال متن: جمعه 8 دیماه 1385

mofidy1
29-12-2006, 14:30
با سلام

دوستان، یک مساله جالب:

فرض کنید


[ برای مشاهده لینک ، لطفا با نام کاربری خود وارد شوید یا ثبت نام کنید ]

حد زیر را محاسبه کنید:


[ برای مشاهده لینک ، لطفا با نام کاربری خود وارد شوید یا ثبت نام کنید ]

موفق باشید.

ali_hp
29-12-2006, 15:04
سلام
دوستان راهنمایی کنند:
با استفاده از فرمول کاردان([ برای مشاهده لینک ، لطفا با نام کاربری خود وارد شوید یا ثبت نام کنید ])
می توان ریشه های یک معادله درجه سوم را به وسیله ترکیباتی از ضرب وجمع و ریشه اعداد گویا وضرایب این معادله و عدد i بیان کرد ایا با حذف i هم این کار امکان پذیر است؟

mofidy1
30-12-2006, 18:18
با سلام

دوستان عزیز، از این به بعد، هر چند هفته یکبار، یک مساله در سطح رشته ریاضی دانشگاه، مطرح خواهم کرد. امیدوارم مورد توجه و استفاده دوستان دانشجو قرار گیرد. البته ممکن است مسائل مطرح شده در این بخش را بتوان با روشهای دبیرستانی هم حل کرد، اما معمولا برای حل آنها نیاز به روشهای پیشرفته تری داریم.

مساله این هفته، یک مساله نسبتاً ساده جبر خطی است.

مساله:

فرض کنید A و B دو ماتریس هم مرتبه باشند به گونه ای که AB=A+B. ثابت کنید AB=BA.

موفق باشید.

ارسال متن: شنبه 9 دیماه 1385

mofidy1
30-12-2006, 18:25
سلام
دوستان راهنمایی کنند:
با استفاده از فرمول کاردان([ برای مشاهده لینک ، لطفا با نام کاربری خود وارد شوید یا ثبت نام کنید ])
می توان ریشه های یک معادله درجه سوم را به وسیله ترکیباتی از ضرب وجمع و ریشه اعداد گویا وضرایب این معادله و عدد i بیان کرد ایا با حذف i هم این کار امکان پذیر است؟

با سلام

دوست عزیز آقای حسین پوران، ظاهرا منظور شما از i ، همان عدد مختلط معروف است. بنده منظور شما را از این جمله متوجه نمی شوم: «ایا با حذف i هم این کار امکان پذیر است؟» لطفا توضیح دهید.

ممنون

aminkarami
30-12-2006, 19:57
سلام
می خواستم ببینم از دوستان کسی هست به من کمک کنه برای حل این مساله:
معکوس تابع:
[ برای مشاهده لینک ، لطفا با نام کاربری خود وارد شوید یا ثبت نام کنید ]

ali_hp
30-12-2006, 21:31
سلام
می خواستم ببینم از دوستان کسی هست به من کمک کنه برای حل این مساله:
معکوس تابع:
[ برای مشاهده لینک ، لطفا با نام کاربری خود وارد شوید یا ثبت نام کنید ]
سلام
اینجا حل شده:[ برای مشاهده لینک ، لطفا با نام کاربری خود وارد شوید یا ثبت نام کنید ]

ali_hp
30-12-2006, 23:17
با سلام

دوست عزیز آقای حسین پوران، ظاهرا منظور شما از i ، همان عدد مختلط معروف است. بنده منظور شما را از این جمله متوجه نمی شوم: «ایا با حذف i هم این کار امکان پذیر است؟» لطفا توضیح دهید.

ممنون
سلام
با استفاده از فرمول کاردان می توان ثابت کرد که ریشه های یک معادله درجه سوم را می توان به صورت ترکیبی از جمع و ضرب و ریشه اعداد مختلط گویا(مجموعه اعداد به صورت x+yi که x,y گویا هستند) و ضرایب معادله بیان کرد به طور دقیقتر می توان گفت:
هرگاهa,b,c,d ضرایب یک معادله درجه سه باشند. ومجموعهS دارای خاصیت های زیر باشد:
یک)نسبت به عمل جمع وضرب بسته است.
دو)اگر a عضوی از Sو n عددی طبیعی باشد انگاه ریشه n ام a نیز عضو S است.
سه) مجموعه اعداد گویا مختلط (اعداد به صورت x+iyکه x,y گویا هستند). زیر مجموعه S است و a,b,c,d نیز از اعضا S هستند.
انگاه ریشه های معادله درجه سوم ما از اعضا S هستند.
حالا سوال من دقیقا این است:
a,b,c,d ضرایب یک معادله درجه سه هستند.مجموعهS دارای خاصیت های زیر است:
یک)زیر مجموعه ای از اعداد حقیقی است.
دو)نسبت به عمل جمع وضرب بسته است.
سه)اگر a عضوی از Sو n عددی طبیعی باشد به طوریکه ریشه n ام عدد a حقیقی باشد. انگاه ریشه n ام a نیز عضو S است.
چهار) مجموعه اعداد گویا زیر مجموعه S است و a,b,c,d نیز از اعضا S هستند.
ایا الزاما می توان نتیجه گرفت که ریشه های حقیقی معادله درجه سوم ما نیز از اعضای S هستند؟

b-z
31-12-2006, 23:29
[ برای مشاهده لینک ، لطفا با نام کاربری خود وارد شوید یا ثبت نام کنید ]

در تقسيم بدون باقيمانده فوق (-) ها نماينده ارقامي هستند كه ممكن است متمايزنباشند . مقسوم و مقسوم عليه و خارج قسمت را تعيين كنيد .
به نظر ساده مياد كسي ميتونه كمك كنه ؟!
كسي نميخواد نظري بده !

ali_hp
31-12-2006, 23:41
سلام دوست عزیز
من یه نیم ساعتی روش فکر کردم ولی به نتیجه ای نرسیدم فکر نمیکنم سوال ساده ای باشه

b-z
01-01-2007, 00:31
سلام دوست عزیز
من یه نیم ساعتی روش فکر کردم ولی به نتیجه ای نرسیدم فکر نمیکنم سوال ساده ای باشه

ممنون از لطف شما
ساير دوستان نظري ندارن ؟
آقاي مفيدي شما يه راه حلي بگين :blush:

m2stech
03-01-2007, 23:13
اقا اين " علم جفر " كه ميگن چيه ؟ واقعا ميشه باهاش آينده را پيش بيني كرد ؟!!!

m_honarmand_j
03-01-2007, 23:48
سلام بر دوستان ریاضی دان
البته من ریاضی دان نیستم و در حد این حرفا نیستم . فقط دوستدار ریاضی ام .
امیدوارم که امتحاناتتونو به خوبی پشت سر بذارید . معلمای ما که محلت نمی دن . حالا بگذریم .
به اتاق ترکیبیات اگه تونستید سری بزنید . یه سری سوال توپ گذاشتم اونجا . در ضمن من منبع برای گراف ازتون خواسته بودم ولی کسی جواب نداد . چرا؟

m_honarmand_j
04-01-2007, 16:32
سلام
ثابت کنید توانی از 2 هست که با 6 تا 9 آغاز می شود .

m_honarmand_j
04-01-2007, 18:23
سلام
خط L و دو نقطه ی A و B در یک صفحه اند . نقطه ای مثل P روی خط L بیابید که از پاره خط های Pa و Pb آنکه بزرگتر است ، کمترین مقدار ممکن باشد .

ali_hp
04-01-2007, 19:11
سلام
با استفاده از فرمول کاردان می توان ثابت کرد که ریشه های یک معادله درجه سوم را می توان به صورت ترکیبی از جمع و ضرب و ریشه اعداد مختلط گویا(مجموعه اعداد به صورت x+yi که x,y گویا هستند) و ضرایب معادله بیان کرد به طور دقیقتر می توان گفت:
هرگاهa,b,c,d ضرایب یک معادله درجه سه باشند. ومجموعهS دارای خاصیت های زیر باشد:
یک)نسبت به عمل جمع وضرب بسته است.
دو)اگر a عضوی از Sو n عددی طبیعی باشد انگاه ریشه n ام a نیز عضو S است.
سه) مجموعه اعداد گویا مختلط (اعداد به صورت x+iyکه x,y گویا هستند). زیر مجموعه S است و a,b,c,d نیز از اعضا S هستند.
انگاه ریشه های معادله درجه سوم ما از اعضا S هستند.
حالا سوال من دقیقا این است:
a,b,c,d ضرایب یک معادله درجه سه هستند.مجموعهS دارای خاصیت های زیر است:
یک)زیر مجموعه ای از اعداد حقیقی است.
دو)نسبت به عمل جمع وضرب بسته است.
سه)اگر a عضوی از Sو n عددی طبیعی باشد به طوریکه ریشه n ام عدد a حقیقی باشد. انگاه ریشه n ام a نیز عضو S است.
چهار) مجموعه اعداد گویا زیر مجموعه S است و a,b,c,d نیز از اعضا S هستند.
ایا الزاما می توان نتیجه گرفت که ریشه های حقیقی معادله درجه سوم ما نیز از اعضای S هستند؟
سلام
اقای مفیدی متوجه منظور من شدید؟

mofidy1
05-01-2007, 12:56
سلام
اقای مفیدی متوجه منظور من شدید؟

با سلام

فرض کنید A مجموعه ای باشد که با اضافه کردن ضرایب یک چند جمله ای ثابت به Q ساخته می شود. آیا منظور شما از S مجموعه ای است که با اضافه کردن جمع و ضرب عناصر A به همراه ریشه های حقیقی عناصر A به A به دست می آید؟

ali_hp
05-01-2007, 14:11
با سلام

فرض کنید A مجموعه ای باشد که با اضافه کردن ضرایب یک چند جمله ای ثابت به Q ساخته می شود. آیا منظور شما از S مجموعه ای است که با اضافه کردن جمع و ضرب عناصر A به همراه ریشه های حقیقی عناصر A به A به دست می آید؟
سلام
نه با این حساب مثلا sqrt{1+sqrt{2}}l عضو S نمی شود.
همین مجموعه A را در نظر بگیرید جمع و ضرب عناصر A به همراه ریشه های حقیقی عناصر A را به مجموعه A اضافه می کنیم.مجموعه بدست امده را B بنامید دوباره جمع و ضرب عناصر Bبه همراه ریشه های حقیقی عناصر B را به B اضافه می کنیم و مجموعه بدست امده را C می نامیم همین کار را برروی C انجام می دهیم و ... این کار را تا بی نهایت انجام می دهیم.مجموعه نهایی مجموعه S است.

mofidy1
05-01-2007, 14:47
با سلام

آقای حسین پوران، لطفا چند جمله ای زیر را در نظر بگیرید و روش ساختن S را برای آن توضیح دهید:


[ برای مشاهده لینک ، لطفا با نام کاربری خود وارد شوید یا ثبت نام کنید ]

موفق باشید.

mofidy1
05-01-2007, 14:56
با سلام

دوستان عزیز، از این به بعد، هر چند هفته یکبار، یک مساله در سطح رشته ریاضی دانشگاه، مطرح خواهم کرد. امیدوارم مورد توجه و استفاده دوستان دانشجو قرار گیرد. البته ممکن است مسائل مطرح شده در این بخش را بتوان با روشهای دبیرستانی هم حل کرد، اما معمولا برای حل آنها نیاز به روشهای پیشرفته تری داریم.

مساله این هفته، یک مساله نسبتاً ساده جبر خطی است.

مساله:

فرض کنید A و B دو ماتریس هم مرتبه باشند به گونه ای که AB=A+B. ثابت کنید AB=BA.

موفق باشید.

ارسال متن: شنبه 9 دیماه 1385

با سلام

مساله را با توجه به این قضیه مهم حل می کنیم که اگر C و D دو ماتریس مربعی باشند و I ماتریس همانی و داشته باشیم CD=I می توان نتیجه گرفت که DC=I.


[ برای مشاهده لینک ، لطفا با نام کاربری خود وارد شوید یا ثبت نام کنید ]

با تشکر از آقای مرتضی بیات، دانشجوی دوره دکتری ریاضی مرکز تحصیلات تکمیلی در علوم پایه زنجان، که این مساله و حل آن، پیشنهاد ایشان بود.

موفق باشید.

ارسال متن: جمعه 15 دیماه 1385

aminkarami
05-01-2007, 18:39
سلام
ببخشید معلم ما اینو سوال داده می خواستم بدونم کسی از دوستان می تونه به من کمک کنه مرسی
[ برای مشاهده لینک ، لطفا با نام کاربری خود وارد شوید یا ثبت نام کنید ]

ali_hp
05-01-2007, 19:46
با سلام

آقای حسین پوران، لطفا چند جمله ای زیر را در نظر بگیرید و روش ساختن S را برای آن توضیح دهید:


[ برای مشاهده لینک ، لطفا با نام کاربری خود وارد شوید یا ثبت نام کنید ]

موفق باشید.
سلام
S را با استفاده از روش زیر می سازیم.که شاید ساده تر باشد:
تعریفی دیگر برای مجموعه S :
یک چند جمله ای درجه 3 داریم به صورت:X^3+ax^2+bx+c .
همه اعداد گویا را به همراه ضرایب چند جمله ای(a,b,c) روی تخته سیاه نوشته ایم.می توانیم حاصلضرب دو عدد که روی تخته سیاه هستندرا روی تخته سیاه بنوسیم و همچنین مجموع دو عدد که روی تخته سیاه هستند را روی تخته سیاه بنویسیم و همچنین ریشه حقیقی هر عدد روی تخته سیاه را روی تخته سیاه می نویسیم.S عبارت است از همه اعدادی که می توانیم روی تخته سیاه بنویسیم.
در مورد چند جمله ای که مثال زدیدضرایب این چند جمله ای یک و b و منفی یک و منفی b هستند که در ان :
[ برای مشاهده لینک ، لطفا با نام کاربری خود وارد شوید یا ثبت نام کنید ]
پس اعدادی که در ابتدا روی تخته نوشته شده اند عبارتند از:همه اعداد گویا و b و منفیb.حالا می خواهیم ببینیم ریشه های این چند جمله ای که یک و پی و معکوس پی هستند روی تخته سیاه ظاهر می شوند یا نه.
داریم:
[ برای مشاهده لینک ، لطفا با نام کاربری خود وارد شوید یا ثبت نام کنید ]

b,-1 روی تخته هستند پس مجموع انها یعنی b-1 را روی تخته می نویسیم.b-1,b-1 روی تخته هستند پس حا صلضرب انها یعنی مجذور b-1 را روی تخته می نویسیم.مجذور b-1 و منفی چهار هر دو را روی تخته هستند
پس مجموع انها یعنی
[ برای مشاهده لینک ، لطفا با نام کاربری خود وارد شوید یا ثبت نام کنید ]
را روی تخته سیاه می نویسیم.به سادگی می توان دید که این عدد مثبت است پس رادیکال ان با فرجه دو را روی تخته سیاه می نویسیم و چون منفی یک روی تخته است حاصلضرب ان با این رادیکال را روی تخته می نویسیم.
b-1 هم روی تخته است پس دو عدد زیر را روی تخته می نویسیم:
[ برای مشاهده لینک ، لطفا با نام کاربری خود وارد شوید یا ثبت نام کنید ]
در نهایت یک دوم هم روی تخته است پس حاصلضرب ان در دو عدد بالا را هم روی تخته می نویسیم.پس توانستیم
پی ومعکوس پی را روی تخته بنویسیم.

ali_hp
05-01-2007, 22:05
سلام
ببخشید معلم ما اینو سوال داده می خواستم بدونم کسی از دوستان می تونه به من کمک کنه مرسی
[ برای مشاهده لینک ، لطفا با نام کاربری خود وارد شوید یا ثبت نام کنید ]
سلام
فرض کنیدp جز اعشاری لگاریتم x باشد و n جز صحیح لگاریتم x. یعنی:
[ برای مشاهده لینک ، لطفا با نام کاربری خود وارد شوید یا ثبت نام کنید ]
پس طبق تعریف لگاریتم داریم:
[ برای مشاهده لینک ، لطفا با نام کاربری خود وارد شوید یا ثبت نام کنید ]
از طرفی:
[ برای مشاهده لینک ، لطفا با نام کاربری خود وارد شوید یا ثبت نام کنید ]
پس داریم:
[ برای مشاهده لینک ، لطفا با نام کاربری خود وارد شوید یا ثبت نام کنید ]

mofidy1
06-01-2007, 00:01
با سلام

آقای حسین پوران، فرض کنید a یک عدد اصم باشد. لطفا برای چند جمله ای زیر هم روش ساختن S را توضیح دهید:


[ برای مشاهده لینک ، لطفا با نام کاربری خود وارد شوید یا ثبت نام کنید ]

متشکرم.

ali_hp
06-01-2007, 04:12
با سلام

آقای حسین پوران، فرض کنید a یک عدد اصم باشد. لطفا برای چند جمله ای زیر هم روش ساختن S را توضیح دهید:


[ برای مشاهده لینک ، لطفا با نام کاربری خود وارد شوید یا ثبت نام کنید ]

متشکرم.
سلام
در مورد این چند جمله ای هم روش ساختن S مشابه است و چیز جدیدی برای گفتن ندارم فقط اینکه نمی توانم مانند مثال قبلی ثابت کنم که ریشه های چند جمله ای از اعضای S هستند. و البته فکر می کنم اگر بتوان ثابت کرد ریشه های این چند جمله ای عضو S هستند به سادگی می توان نتیجه رابه هر چند جمله ای به صورتx^3+cx^2+dx+e تعمیم داد.

mofidy1
06-01-2007, 08:15
سلام
در مورد این چند جمله ای هم روش ساختن S مشابه است و چیز جدیدی برای گفتن ندارم فقط اینکه نمی توانم مانند مثال قبلی ثابت کنم که ریشه های چند جمله ای از اعضای S هستند. و البته فکر می کنم اگر بتوان ثابت کرد ریشه های این چند جمله ای عضو S هستند به سادگی می توان نتیجه رابه هر چند جمله ای به صورتx^3+cx^2+dx+e تعمیم داد.

آفرین، هدف بنده هم دقیقا همین بود. ظاهرا بر اساس ایده های شما در پست قبلی، می توان ثابت کرد که اگر حداقل یکی از این ریشه ها گویا باشد، ادعای شما در مورد S درست است. حال اگر هر سه اصم باشند چه؟

موفق باشید.

ali_hp
06-01-2007, 17:04
آفرین، هدف بنده هم دقیقا همین بود. ظاهرا بر اساس ایده های شما در پست قبلی، می توان ثابت کرد که اگر حداقل یکی از این ریشه ها گویا باشد، ادعای شما در مورد S درست است. حال اگر هر سه اصم باشند چه؟

موفق باشید.
در حقیقت اگر ما بتوانیم ثابت کنیم یکی از ریشه های چند جمله ای مثلا q عضوی از S است انگاه با تقسیم چند جمله ای بر x-q و استفاده از فرمول معادله درجه دوم برای خارج قسمت تقسیم می توان نشان داد بقیه ریشه های حقیقی هم عضو S هستند.در حالت خاص وقتی یک ریشه چند جمله ای عدد گو یا باشد به وضو ح این عدد گویاعضو S است و می توان نشان داد که بقیه ریشه های حقیقی نیز عضو S هستند.
در مورد این چند جمله ای که ریشه هایش عدد پی و عدد نپر بودند من ادعایم را در مورد S نتوانستم ثابت کنم ولی
هنوز معلوم نیست که درست است یا غلط.در حقیقت من این ادعا را در مورُد S در حالت کلی ثابت نکرده ام. و مثال نقضی هم برای ان پیدا نکرده ام.
این ادعا در مورد S را در حالتهای خاص زیادی ثابت کرده ام که یکی از کلی ترین انها چند جمله ای های با خا صیت زیر هستند:
همه چند جمله ای های به صورت x^3+cx^2+dx+e که پس از تغییر متغیر X=Y-c/3 و باز نویسی به شکل
Y^3+pY+q (که p,q بر حسب c,d,e مشخص می شوند.) دا شته باشیم:
[ برای مشاهده لینک ، لطفا با نام کاربری خود وارد شوید یا ثبت نام کنید ]
که البته اثبات ان با استفاده از فرمول کاردان و انچه در با لا امده است ساده است.

mofidy1
06-01-2007, 17:33
با سلام

یک چهار ضلعی محدب را در نظر گرفته، دو قطر آن را رسم می کنیم. اگر محیطهای چهار مثلث ایجاد شده مساوی باشند، ثابت کنید که این چهار ضلعی لوزی است.

موفق باشید.

ارسال متن: شنبه 16 دیماه 1385

imannima
11-01-2007, 00:40
ببخشيد يك سؤال دارم
اگر وتري از يك دايره به قطر 10 و طول وتر 8 باشد چگونه ميتوان مساحت قسمتي از دايره كه به وسيله وتر جدا ميشود را محاسبه كرد؟
آيا براي اينكار فرمولي هست؟
آيا نياز به معلومات بيشتري هست ؟ مثلا اندازه زاويه روبرو به وتر
لطف ميكنيد اگر پاسخ دهيد.

m_honarmand_j
11-01-2007, 01:30
سلام
مهندسان همسشه راست می گویند و بازرگانان دروغ . F و G مهندس اند . A توضیح می دهد ، که B تاکید دارد ، که C باور دارد ، که D می گوید ، که E اصرار می کند که F مهندس بودن G را نفی می کند . C هم توضیح می دهد که D بازر گان است . اگر A بازر گان باشد ، روی هم رفته در اینجا چند بازر گان وجود دارد؟

ali_hp
11-01-2007, 01:59
ببخشيد يك سؤال دارم
اگر وتري از يك دايره به قطر 10 و طول وتر 8 باشد چگونه ميتوان مساحت قسمتي از دايره كه به وسيله وتر جدا ميشود را محاسبه كرد؟
آيا براي اينكار فرمولي هست؟
آيا نياز به معلومات بيشتري هست ؟ مثلا اندازه زاويه روبرو به وتر
لطف ميكنيد اگر پاسخ دهيد.
سلام
فرض کنید در دایره ای به شعاع R و مرکز O ; وتر AB به طول L داده شده است.داریم:
مساحت ناحیه جدا شده توسط وتر= (مساحت قطاع OAB )منهای(مساحت مثلث OAB)
پس اگر زاویه رو برو به وتر برابر C (بر حسب رادیان)باشد مساحت ناحیه جدا شده توسط وتر از رابطه زیر بدست می اید:
[ برای مشاهده لینک ، لطفا با نام کاربری خود وارد شوید یا ثبت نام کنید ]
زاویه C به بر حسب L و R مشخص می شود در حقیقت اگر عمود OH را بر AB فرود بیاوریم با توجه به اینکه OH زاویه AOB را نصف می کند داریم:
[ برای مشاهده لینک ، لطفا با نام کاربری خود وارد شوید یا ثبت نام کنید ]

m_honarmand_j
11-01-2007, 03:04
سلام
به ازای چه مقدار n عدد 1+(2به توانn )برابر توانی از یک عدد طبیعی است؟

imannima
11-01-2007, 06:51
آقاي ali_hp بسيار ممنونم
لطف بزرگي فرموديد
انشاءالله هميشه موفق و سربلند باشيد

ali_hp
11-01-2007, 12:20
آقاي ali_hp بسيار ممنونم
لطف بزرگي فرموديد
انشاءالله هميشه موفق و سربلند باشيد
خواهش می کنم.فقط اون رابطه مربوط به مساحت ناحیه جدا شده توسط وتر درست نبود.چون ضریب یک دوم مربوط به مساحت قطاع دایره جا افتاده بود .که رابطه رو ویزایش کردم والان درسته.

m_honarmand_j
11-01-2007, 17:55
سلام
در مربعی ، یک مستطیل (که طول و عرض نا برابر دارد ) را محاط کرده ایم . ثابت کنید مجموع طول و عرض (نصف محیط مستطیل) برابر است با طول قطر مربع .

mofidy1
12-01-2007, 12:34
با سلام

یک چهار ضلعی محدب را در نظر گرفته، دو قطر آن را رسم می کنیم. اگر محیطهای چهار مثلث ایجاد شده مساوی باشند، ثابت کنید که این چهار ضلعی لوزی است.

موفق باشید.

ارسال متن: شنبه 16 دیماه 1385

با سلام

می توانیم فرض کنیم که AO کمتر یا مساوی OC و BO کمتر یا مساوی OD است. اگر M و N دو نقطه روی OC و OD باشند به طوریکه AO=OM و BO=ON، آنگاه ABMN متوازی الاضلاع است. چون محیط ABO و MNO یکی است، لذا دو مثلث CDO و MNO برابرند و لذا M=C و N=D و در نتیجه ABCD متوازی الاضلاع است. حال چون محیطهای ABO و BCO برابرند، پس AB=BC که مطلب را ثابت می کند.

موفق باشید.

ارسال متن: جمعه 22 دیماه 1385

m_honarmand_j
12-01-2007, 16:38
جواب سوال مهندسان و بازر گانان
اگر مهندس را با م و بازر گان را با ب نشان دهیم توزیع حروف در گروه Abcdefg ، یکی از این چهار حالت است .
م م ب ب م م ب ، م م ب م ب م ب ، م م م ب م ب ب ، م م م م ب ب ب .
پس جواب می شود سه .
(دوستان به اتاق ترکیبیات هم سر بزنید)

mofidy1
13-01-2007, 13:32
با سلام

ثابت کنید حاصل ضرب n عدد صحیح متوالی بر !n بخشپذیر است.

موفق باشید.

ارسال متن: شنبه 23 دیماه 1385

edwardfurlong
14-01-2007, 07:30
اگر ممکن است هم ارزی های مهم برای کنکور را به e-mail من ارسال کنید.



با تشکر edwardfurlong
voldomort731@yahoo.com

aminkarami
14-01-2007, 13:26
با سلام

ثابت کنید حاصل ضرب n عدد صحیح متوالی بر !n بخشپذیر است.

موفق باشید.

ارسال متن: شنبه 23 دیماه 1385

سلام
نمی دونم راهم درسته یانه:
می دانیم که حاصل ضرب هر n عدد متوالی n را عاد می کند پس می توان حاصلضرب n عدد را به صورت مجموعه ای از 1،2،3و...،n-1؛n افراز کرد که هر که مثلا n-1 عدد می تواند n-1 را عا کند و... پس می توانیم بگوییم n! را عاد می کند.
مرسی

behnam karami
15-01-2007, 17:33
یه سوال دارم اگه حلش کنید خیلی ممنون میشم.
شکل زیر مفروض است جمع زاویه های A,B,C,D,Eرا بیابیدC:\Documents and Settings\behnam\My Documents\My Pictures\untitled

Vmusic
15-01-2007, 17:52
یه سوال دارم اگه حلش کنید خیلی ممنون میشم.
شکل زیر مفروض است جمع زاویه های A,B,C,D,Eرا بیابیدC:\Documents and Settings\behnam\My Documents\My Pictures\untitled

با سلام خدمت شما دوست عزیز

شما می توانید تصویر مورد نظرتون رو تو سایتای اپلود عکس اپ کنید بعد لینک مورد نظر رو قرار بدید

مثل tinypic.com


[ برای مشاهده لینک ، لطفا با نام کاربری خود وارد شوید یا ثبت نام کنید ]

eh_mn
15-01-2007, 19:56
B3. فرض کنید R حلقه ای با مشخصه صفر باشد (R لزوما تعویضپذیر نیست). فرض کنید f،e و g عناصر خودتوان R باشند به طوریکه مجموع آنها صفر است. ثابت کنید که باید هر سه صفر باشند.

(اين سوال با كمك يكي از دوستان حل شده است)
داريم
[ برای مشاهده لینک ، لطفا با نام کاربری خود وارد شوید یا ثبت نام کنید ]
يعني
[ برای مشاهده لینک ، لطفا با نام کاربری خود وارد شوید یا ثبت نام کنید ]
با ضرب طرفين رابطه (1) از سمت چپ و راست در a داريم
[ برای مشاهده لینک ، لطفا با نام کاربری خود وارد شوید یا ثبت نام کنید ]
و لذا
[ برای مشاهده لینک ، لطفا با نام کاربری خود وارد شوید یا ثبت نام کنید ]
با جايگذاري رابطه اخير در (1) و ضرب طرفين در a نتيجه مي شود كه b=c. به همين ترتيب مي توان نشان داد كه a=b. ولذا بنابه فرض 3a=0. پسa=b=c=0.

edwardfurlong
17-01-2007, 08:47
tgx>sinx>x
در نتیجه sinx/x بین 1 و tgx/x قرار میگیرد . وقتی 0<----x حد tgx/x برابر 1 می شود.
طبق قضیه ساندویچ حد sinx/x نیز 1 می شود.

ali_hp
17-01-2007, 16:51
tgx>sinx>x

سلام
فکر می کنم منظورت نامساوی زیر باشه برای x بین صفر و پی دوم:
sinx<x<tgx

mofidy1
17-01-2007, 21:29
tgx>sinx>x
در نتیجه sinx/x بین 1 و tgx/x قرار میگیرد . وقتی 0<----x حد tgx/x برابر 1 می شود.
طبق قضیه ساندویچ حد sinx/x نیز 1 می شود.

با سلام

دوست عزیز، ورودتان را به اتاق ریاضیات خوش آمد می گوییم. لطفا به این سوال پاسخ دهید: چرا وقتی 0<----x حد tgx/x برابر 1 می شود؟ آیا روشی غیر از روش مطرح شده در حسابان برای این مساله می دانید؟

موفق باشید.

m_honarmand_j
18-01-2007, 12:14
سلام
ثابت کنید مجموع فاصله های یک نقطه ی درونی چهار وجهی تا رأس های آن از محیط چهار وجهی کمتر است .
(به اتاق ترکیبیات هم سری بزنید ، خوشحال می شم)

m_honarmand_j
18-01-2007, 12:19
سلام
روی هر ضلع مثلث ، نثطه ای انتخاب و آن ها را به هم وصل کرده ایم . به این ترتیب ، چهار مثلثکوچک بدست می آید . می دانیم این چهار مثلث محیط هایی برابر دارند . ثابت کنید نقطه های انتخابی در وسط ضلع ها واقع اند .
(به اتاق تر کیبیات هم سری بزنید )

mofidy1
19-01-2007, 17:45
با سلام

ثابت کنید حاصل ضرب n عدد صحیح متوالی بر !n بخشپذیر است.

موفق باشید.

ارسال متن: شنبه 23 دیماه 1385

با سلام

از aminkarami که در پست 611 به حل مساله پرداختند تشکر می کنم. برای حل این مساله توجه کنید که اگر یکی از این اعداد صفر باشند، مطلب بدیهی است؛ لذا فرض می کنیم که همه اعداد ناصفر باشند و در نتیجه همه مثبت یا همه منفی هستند. می توان فرض کرد همه اعداد مثبت هستند(در غیر اینصورت کافی است قدر مطلق حاصل ضرب را در نظر بگیریم). می دانیم که


[ برای مشاهده لینک ، لطفا با نام کاربری خود وارد شوید یا ثبت نام کنید ]

که طرف چپ، تعدادانتخابهایn شیء از میان m+n شیء است، که عددی طبیعی است و لذا تساوی بالا حل مساله را کامل می کند.

موفق باشید.

ارسال متن: جمعه 29 دیماه 1385

mofidy1
20-01-2007, 09:22
با سلام

مجموعه ای n+1 عضوی از اعداد طبیعی و نابیشتر از 2n را در نظر بگیرید. ثابت کنید حداقل یک عضو این مجموعه، عضو دیگری از آن را می شمارد.

موفق باشید.

ارسال متن: شنبه 30 دیماه 1385

b-z
22-01-2007, 21:18
[ برای مشاهده لینک ، لطفا با نام کاربری خود وارد شوید یا ثبت نام کنید ]

در تقسيم بدون باقيمانده فوق (-) ها نماينده ارقامي هستند كه ممكن است متمايزنباشند . مقسوم و مقسوم عليه و خارج قسمت را تعيين كنيد .
به نظر ساده مياد كسي ميتونه كمك كنه ؟!

شرمنده كه دوباره كاري ميكنم از دوستان كسي نظري راجع به اين مساله نداره .
آقاي مفيدي نظرتون رو نميگين ؟!

mofidy1
23-01-2007, 19:28
شرمنده كه دوباره كاري ميكنم از دوستان كسي نظري راجع به اين مساله نداره .
آقاي مفيدي نظرتون رو نميگين ؟!

با سلام

حال که اصرار دارید، به روی چشم.

مقسوم عبارت است از 7375428413

مقسوم علیه عبارت است از: 125473

خارج قسمت: 58781

با تقسیم معمولی بقیه جاهای خالی نیز به دست می آید. این مساله به «مساله برویک» معروف است. راه حل آن بسیار طولانی است. اگر به راه حل آن نیاز دارید به کتاب «فنون مساله حل کردن» تالیف «استیون ج. کرانتس» و ترجمه «مهران اخباریفر» (انتشارات فاطمی) صفحه 165 مراجعه فرمایید.

موفق باشید.

b-z
26-01-2007, 02:46
با سلام

حال که اصرار دارید، به روی چشم.

مقسوم عبارت است از 7375428413

مقسوم علیه عبارت است از: 125473

خارج قسمت: 58781

با تقسیم معمولی بقیه جاهای خالی نیز به دست می آید. این مساله به «مساله برویک» معروف است.

موفق باشید.

استاد عزيز اقاي مفيدي ممنون از جوابتون

نميخواين روش حل مساله رو توضيح بدين ؟

mofidy1
26-01-2007, 16:53
ممنون از جوابتون
نميخواين روش حل مساله رو توضيح بدين ؟

با سلام

دوست عزیز راه حل آن بسیار طولانی است و آوردن آن در اینجا مناسب نیست. اگر به راه حل آن نیاز دارید به کتاب «فنون مساله حل کردن» تالیف «استیون ج. کرانتس» و ترجمه «مهران اخباریفر» (انتشارات فاطمی) صفحه 165 مراجعه فرمایید.

موفق باشید.

mofidy1
26-01-2007, 20:04
با سلام

مجموعه ای n+1 عضوی از اعداد طبیعی و نابیشتر از 2n را در نظر بگیرید. ثابت کنید حداقل یک عضو این مجموعه، عضو دیگری از آن را می شمارد.

موفق باشید.

ارسال متن: شنبه 30 دیماه 1385

با سلام

فرض کنید


[ برای مشاهده لینک ، لطفا با نام کاربری خود وارد شوید یا ثبت نام کنید ]

n+1 عدد طبیعی باشند که همگی کمتر یا مساوی 2n هستند. برای هر i می نویسیم:


[ برای مشاهده لینک ، لطفا با نام کاربری خود وارد شوید یا ثبت نام کنید ]

که در آن n_i عددی صحیح و نامنفی و y_i فرد است. بنابراین y_i ها تشکیل مجموعه ای شامل حداکثر n+1 عدد فرد کمتر از 2n می دهند. چون فقط n عدد فرد کمتر از 2n وجود دارد لذا بنابر اصل لانه کبوتری (اصل حجره ها) باید برای حداقل یک i و j (که متمایزند) داشته باشیم: y_i=y_j. بنابراین یا x_i ، x_j را عاد می کند یا بالعکس.

موفق باشید.

ارسال متن: جمعه 6 بهمن 1385

mofidy1
27-01-2007, 23:47
با سلام

مساله ساده اما بسیار معروف زیر را حل کنید:

تابع f با شرط زیر را در نظر بگیرید:


[ برای مشاهده لینک ، لطفا با نام کاربری خود وارد شوید یا ثبت نام کنید ]

ثابت کنید برای هر عدد گویای x داریم:


[ برای مشاهده لینک ، لطفا با نام کاربری خود وارد شوید یا ثبت نام کنید ]

موفق باشید.

ارسال متن: شنبه 7 بهمن 1385

m_honarmand_j
28-01-2007, 23:41
سلام
20 دانش آموز ، برای حل کردن 20 مسئله ، گرد هم آمدند . هر دانش آموز دو مسئله را حل کرد و هر مسئله به وسیله ی دو دانش آموز حل شد . ثابت کنید می توان ترتیبی داد که هر دانش آموز راه حل یکی از مسئله ها را بیان کند ، به نحوی که همه ی مسئله ها توضیح داده شده باشند .
(دوستان به اتاق ترکیبیات هم سر بزنید . پشیمون نمی شید)

mofidy1
29-01-2007, 08:34
با سلام

دوستان عزیز مساله زیر را همراه با حلش، به تقاضای یکی از اعضای جدید انجمن ریاضیات- ironroad - خد متتان تقدیم می کنم:

مساله: ثابت کنید


[ برای مشاهده لینک ، لطفا با نام کاربری خود وارد شوید یا ثبت نام کنید ]

حل مساله:

قرار دهید


[ برای مشاهده لینک ، لطفا با نام کاربری خود وارد شوید یا ثبت نام کنید ]

و توجه کنید که


[ برای مشاهده لینک ، لطفا با نام کاربری خود وارد شوید یا ثبت نام کنید ]

از طرف دیگر a_i ها ریشه های معادله sin7x=0 هستند؛ اما می دانیم که


[ برای مشاهده لینک ، لطفا با نام کاربری خود وارد شوید یا ثبت نام کنید ]

لذا چون a_0=0 بنابر قضیه ویت (Vieta's Theorem) داریم:


[ برای مشاهده لینک ، لطفا با نام کاربری خود وارد شوید یا ثبت نام کنید ]

حال قرار دهید:


[ برای مشاهده لینک ، لطفا با نام کاربری خود وارد شوید یا ثبت نام کنید ]

بنابر این:


[ برای مشاهده لینک ، لطفا با نام کاربری خود وارد شوید یا ثبت نام کنید ]

که نتیجه را به دست می دهد.

موفق باشید.

m_honarmand_j
29-01-2007, 19:31
سلام
20 دانش آموز ، برای حل کردن 20 مسئله ، گرد هم آمدند . هر دانش آموز دو مسئله را حل کرد و هر مسئله به وسیله ی دو دانش آموز حل شد . ثابت کنید می توان ترتیبی داد که هر دانش آموز راه حل یکی از مسئله ها را بیان کند ، به نحوی که همه ی مسئله ها توضیح داده شده باشند .
(دوستان به اتاق ترکیبیات هم سر بزنید . پشیمون نمی شید)

سلام
اگر با قضیه فیلیپ هال آشنایی داشته باشید می بینید که به راحتی با اون قضیه حل می شود . قضیه هال به قضیه ازدواج نیز معروف است .

aminpi
29-01-2007, 20:00
با سلام خدمت دوستان و یاران!
می خواستم بدونم که انتگرال زیر چجوری حل میشه. من اوّل خواستم همین انتگرال رو بدون جمله x با روش تغییر متغیر مثلثاتی یا نصف کمان حل کنم که بعدش بتونم با جزء به جزء به جواب برسم، ولی نشد. لطفاً زاهنمایی ام کنید. با تشکر...


[ برای مشاهده لینک ، لطفا با نام کاربری خود وارد شوید یا ثبت نام کنید ]

aminpi
30-01-2007, 20:41
از دوستان کسی پیدا نمیشه در مورد سوال ما نظری بده؟؟؟؟

mofidy1
31-01-2007, 19:16
با سلام خدمت دوستان و یاران!
می خواستم بدونم که انتگرال زیر چجوری حل میشه. من اوّل خواستم همین انتگرال رو بدون جمله x با روش تغییر متغیر مثلثاتی یا نصف کمان حل کنم که بعدش بتونم با جزء به جزء به جواب برسم، ولی نشد. لطفاً راهنمایی ام کنید. با تشکر...


[ برای مشاهده لینک ، لطفا با نام کاربری خود وارد شوید یا ثبت نام کنید ]

با سلام


[ برای مشاهده لینک ، لطفا با نام کاربری خود وارد شوید یا ثبت نام کنید ]

موفق باشید.

mofidy1
03-02-2007, 14:42
با سلام

مساله ساده اما بسیار معروف زیر را حل کنید:

تابع f با شرط زیر را در نظر بگیرید:


[ برای مشاهده لینک ، لطفا با نام کاربری خود وارد شوید یا ثبت نام کنید ]

ثابت کنید برای هر عدد گویای x داریم:


[ برای مشاهده لینک ، لطفا با نام کاربری خود وارد شوید یا ثبت نام کنید ]

موفق باشید.

ارسال متن: شنبه 7 بهمن 1385

با سلام

فرض کنید m و n اعداد طبیعی و s عددی صحیح و ناصفر باشند. مراحل مختلف حل مساله را در لینک زیر مطالعه فرمایید:

[ برای مشاهده لینک ، لطفا با نام کاربری خود وارد شوید یا ثبت نام کنید ]

موفق باشید.

ارسال متن: شنبه 14 بهمن 1385

mofidy1
03-02-2007, 17:15
با سلام

نا مساوی پرکاربرد زیر، به نامساوی هندسی-حسابی معروف است. سعی کنید آنرا فقط با استفاده از ابزار هندسی «ثابت» کنید. (توجه کنید که a و b اعداد حقیقی مثبت هستند.)


[ برای مشاهده لینک ، لطفا با نام کاربری خود وارد شوید یا ثبت نام کنید ]

موفق باشید.

ارسال متن: شنبه 14 بهمن 1385

attractive_girl
03-02-2007, 23:55
سلام
روش حساب کردن عدد طلايي رو ميخوام .اگه زودتر جوابش رو بدین ممنون میشم.
فقط خواهشا اگه کسی میدونه بگه جوابش رو که نمرم نپـــره:rolleye:

ali1234
04-02-2007, 17:34
سلام آقاي مفيدي
در حل مسئله هفته قبل آيا نوشتن مراحل 2 و3 لازم است ؟

Iron
04-02-2007, 23:36
با سلام

نا مساوی پرکاربرد زیر، به نامساوی هندسی-حسابی معروف است. سعی کنید آنرا فقط با استفاده از ابزار هندسی «ثابت» کنید. (توجه کنید که a و b اعداد حقیقی مثبت هستند.)


[ برای مشاهده لینک ، لطفا با نام کاربری خود وارد شوید یا ثبت نام کنید ]

موفق باشید.

ارسال متن: شنبه 14 بهمن 1385

[ برای مشاهده لینک ، لطفا با نام کاربری خود وارد شوید یا ثبت نام کنید ]

mofidy1
05-02-2007, 20:37
سلام آقاي مفيدي
در حل مسئله هفته قبل آيا نوشتن مراحل 2 و3 لازم است ؟

نه لزومی ندارند؛ اما برای فهم بهتر مراحل بعد - آنهم برای دانش آموزان - نوشتن این مراحل خالی از فایده نیست. از دقت نظرتان متشکرم.

موفق باشید.

ali1234
05-02-2007, 23:53
با سلام
آقاي مفيدي شايد جاي اين سوال اينجا نباشد ولي مي خواستم به من بگوييد چه شغلي براي يك ليسانس رياضي محض در نظر گرفته شده است؟
با تشكر

m_honarmand_j
06-02-2007, 18:20
سلام
ثابت کنید جمع مربعات پنج عدد طبیعی متوالی مربع کامل نیست .
(به اتاق ترکیبیات هم سر بزنید)

m_honarmand_j
06-02-2007, 18:22
ثابت کنید اگر n حاصل جمع دو مربع کامل باشد 2n نیز چنین است .
(به اتاق ترکیبات هم یر بزنید)

mofidy1
10-02-2007, 16:04
با سلام

نا مساوی پرکاربرد زیر، به نامساوی هندسی-حسابی معروف است. سعی کنید آنرا فقط با استفاده از ابزار هندسی «ثابت» کنید. (توجه کنید که a و b اعداد حقیقی مثبت هستند.)


[ برای مشاهده لینک ، لطفا با نام کاربری خود وارد شوید یا ثبت نام کنید ]

موفق باشید.

ارسال متن: شنبه 14 بهمن 1385

با سلام

از Iron که مساله را در پست 638 ([ برای مشاهده لینک ، لطفا با نام کاربری خود وارد شوید یا ثبت نام کنید ])حل کردند متشکرم. راه حل ایشان را در لینک زیر ببینید:

[ برای مشاهده لینک ، لطفا با نام کاربری خود وارد شوید یا ثبت نام کنید ]

راه حل دیگری را خدمتتان ارائه می کنم. به شکل زیر توجه کنید. مرکز دایره ها B و E است. قطر دایره های بزرگ و کوچک را به ترتیب a و b فرض کنید. با استفاده از قضیه نامساوی مثلث مساله حل خواهد شد.


[ برای مشاهده لینک ، لطفا با نام کاربری خود وارد شوید یا ثبت نام کنید ]


لازم به ذکر است که روشهای دیگر اثبات هندسی این مساله را می توانید در کتاب زیبای «اثبات بدون کلام» ترجمه سپیده چمن آرا مطالعه فرمایید.

موفق باشید.

ارسال متن: شنبه 21 بهمن 1385

mofidy1
10-02-2007, 16:36
با سلام

بدون استفاده از قضیه فیثاغورث اتحاد مثلثاتی معروف زیر را ثابت کنید!!


[ برای مشاهده لینک ، لطفا با نام کاربری خود وارد شوید یا ثبت نام کنید ]

موفق باشید.

ارسال متن: شنبه 21 بهمن 1385

behnam karami
11-02-2007, 22:26
آقای مفیدی من نمیتونم فا یلشو آپلود کنم .لطفا راهنمایی کنید

behnam karami
12-02-2007, 13:44
با سلام

بدون استفاده از قضیه فیثاغورث اتحاد مثلثاتی معروف زیر را ثابت کنید!!


[ برای مشاهده لینک ، لطفا با نام کاربری خود وارد شوید یا ثبت نام کنید ]

موفق باشید.

ارسال متن: شنبه 21 بهمن 1385

این سوال راه حل های زیادی داره .ولی من ساده ترینش رو می نویسم .فقط من نتونستم حرف آلفا رو تایپ کنم به خاطر همین ازحرف a استفاده می کنم

sin^2a=sina*sina
cos^2a=cosa*cosa
sin^2a+cos^2a=sina*sina+cosa*cosa=cos(a-a)=cos0=1

eh_mn
14-02-2007, 01:48
این سوال راه حل های زیادی داره .ولی من ساده ترینش رو می نویسم .فقط من نتونستم حرف آلفا رو تایپ کنم به خاطر همین ازحرف a استفاده می کنم

sin^2a=sina*sina
cos^2a=cosa*cosa
sin^2a+cos^2a=sina*sina+cosa*cosa=cos(a-a)=cos0=1
سلام
تساوي sina*sina+cosa*cosa=cos(a-a)l رو چطور بدون استفاده از رابطه فيثاغورث ثابت مي‌كنيد؟

ali1234
15-02-2007, 11:55
با سلام
آقاي مفيدي شايد جاي اين سوال اينجا نباشد ولي مي خواستم به من بگوييد چه شغلي براي يك ليسانس رياضي محض در نظر گرفته شده است؟
با تشكر

با سلام
كسي نمي خواد به اين سوال جواب بده؟!!

mofidy1
17-02-2007, 16:50
با سلام

بدون استفاده از قضیه فیثاغورث اتحاد مثلثاتی معروف زیر را ثابت کنید!!


[ برای مشاهده لینک ، لطفا با نام کاربری خود وارد شوید یا ثبت نام کنید ]

موفق باشید.

ارسال متن: شنبه 21 بهمن 1385

با سلام

با تشکر از behnam karami که در پست 646 ([ برای مشاهده لینک ، لطفا با نام کاربری خود وارد شوید یا ثبت نام کنید ]) مساله را به زیبایی حل کردند. راه حل ایشان کاملا درست است و اتحادی که ایشان استفاده کرده اند در کتاب ریاضی 2 ی دبیرستان بدون استفاده از قضیه فیثاغورث ثابت شده است.

راه حل دیگری را با استفاده از مشتق خدمتتان تقدیم می کنم:

می دانیم که اگر مشتق تابعی در یک بازه صفر شود، آن تابع در آن بازه ثابت است. لذا می توان با محاسبه تابع فقط در یک نقطه، مقدار ثابت را یافت؛ بنابر این:


[ برای مشاهده لینک ، لطفا با نام کاربری خود وارد شوید یا ثبت نام کنید ]

موفق باشید.

ارسال متن: شنبه 28 بهمن 1385

mofidy1
17-02-2007, 19:22
با سلام

شکل یک چند جمله ای درجه ی 3 را که دارای یک ماکزیمم و یک می نیمم است، در نظر بگیرید. همه خطوطی را در نظر بگیرید که این شکل را در سه نقطه قطع می کنند. ثابت کنید مجموع طولهای این سه نقطه ثابت است و بستگی به خط ندارد. (راهنمایی: با مساوی قرار دادن فرمول خط و تابع درجه ی سه به یک معادله برسید و رابطه بین ریشه را در نظر بگیرید.)

آیا می توانید این مساله را تعمیم دهید؟

موفق باشید.

ارسال متن: شنبه 28 بهمن 1385

ali_hp
19-02-2007, 18:56
سلام
فکر نمی کنم سوال ساده ای باشه ولی به نظر من خیلی جالبه:
فرض کنید P چند جمله ای با ضرایب صحیح باشدوضریب پیشرو ان یک باشد وd عددی طبیعی به طوریکه درجه P برd بخش پذیر باشد و معادله
[ برای مشاهده لینک ، لطفا با نام کاربری خود وارد شوید یا ثبت نام کنید ]
نا متناهی جواب برای m,n طبیعی داشته باشد.ثابت کنید چند جمله ای Q با ضرایب صحیح وجود دارد به طوری که برای هر x:
[ برای مشاهده لینک ، لطفا با نام کاربری خود وارد شوید یا ثبت نام کنید ]
سلام
تعمیم این مساله:
فرض کنید P و G دو چند جمله ای با ضرایب گویای و ضریب پیشرو مثبت باشند و درجهG برابر d باشد(d عددی طبیعی است)و درجه P برابر مضربی ناصفر از d باشد.همچنین نسبت ضریب پیشرو P به ضریب پیشرو G برابر توان d ام یک عدد گویا باشد. ثابت کنید:

معادله[ برای مشاهده لینک ، لطفا با نام کاربری خود وارد شوید یا ثبت نام کنید ] دارای نامتناهی جواب برای اعداد طبیعی m,n است اگر و تنها اگر چند جمله ای Q با ضرایب گویا و ضریب پیشرو مثبت موجود باشد به طوریکه برای هر x :

[ برای مشاهده لینک ، لطفا با نام کاربری خود وارد شوید یا ثبت نام کنید ]

ali_hp
19-02-2007, 20:09
سلام
فکر نمی کنم سوال ساده ای باشه ولی به نظر من خیلی جالبه:
فرض کنید P چند جمله ای با ضرایب صحیح باشدوضریب پیشرو ان یک باشد وd عددی طبیعی به طوریکه درجه P برd بخش پذیر باشد و معادله
[ برای مشاهده لینک ، لطفا با نام کاربری خود وارد شوید یا ثبت نام کنید ]
نا متناهی جواب برای m,n طبیعی داشته باشد.ثابت کنید چند جمله ای Q با ضرایب صحیح وجود دارد به طوری که برای هر x:
[ برای مشاهده لینک ، لطفا با نام کاربری خود وارد شوید یا ثبت نام کنید ]
سلام
تعمیم این مساله:
فرض کنید P و G دو چند جمله ای با ضرایب گویای و ضریب پیشرو مثبت باشند و درجهG برابر d باشد(d عددی طبیعی است)و درجه P برابر مضربی ناصفر از d باشد.همچنین نسبت ضریب پیشرو P به ضریب پیشرو G برابر توان d ام یک عدد گویا باشد. ثابت کنید:

معادله[ برای مشاهده لینک ، لطفا با نام کاربری خود وارد شوید یا ثبت نام کنید ] دارای نامتناهی جواب برای اعداد طبیعی m,n است اگر و تنها اگر چند جمله ای Q با ضرایب گویا و ضریب پیشرو مثبت موجود باشد به طوریکه برای هر x :

[ برای مشاهده لینک ، لطفا با نام کاربری خود وارد شوید یا ثبت نام کنید ]

m_honarmand_j
20-02-2007, 04:26
سلام
22 نقطه را روی بازه ی [0,1] علامت گذاشته ایم . می توانیم ، هر دو نقطه را ، با نقطه ی وسط پاره خط راستی که آنها را به هم وصل می کند عوض کنیم . ثابت کنید ، اگر 20 بار این عمل را انجام دهیم ، به جایی می رسیم که ، فاصله ی دو نقطه ی باقی مانده ، از 001/0 تجاوز نمی کند .
( دوستان به اتاق ترکیبیات هم سر بزنید)

mss_sdt
20-02-2007, 23:03
سلام به همه دوستان و استادان ریاضی
یه سوال امتیازی مطرح شده که 2 نمره داره ، هر کی زودتر بده ، نمره رو میگیره ، پس لطفا سریع کمک کنید.

طریقه حل ضرب خارجی در فضای 4 بعدی (یعنی به جای 3 متغیر ، 4 متغیر داشته باشه)
مثلا (a,b,c,d)*(x,y,z,g) ستاره به جای علامت ضرب خارجی به کار رفته.

لطفا روش حل کردن ضرب خارجی بالا رو به من بگید.(فقط زودتر ، تا 2 نمره بگیرم ، 2 نمره تو ریاضی 2 خیلیه;) )
با تشکر از همه دوستان

attractive_girl
21-02-2007, 16:01
سلام اگه ميشه تا شنبه جواب سوال منو بدين ممنون ميشم.فقط نامردي نکنين ها جوابم رو بدين.ممنون
sin nx & cos nx را حساب کنيد

ali_hp
21-02-2007, 20:42
سلام اگه ميشه تا شنبه جواب سوال منو بدين ممنون ميشم.فقط نامردي نکنين ها جوابم رو بدين.ممنون
sin nx & cos nx را حساب کنيد
سلام
با استفاده از فرمول مواور و بسط دو جمله ای می توان روابط زیر را بدست اورد:
[ برای مشاهده لینک ، لطفا با نام کاربری خود وارد شوید یا ثبت نام کنید ]

mss_sdt
21-02-2007, 21:45
دوستان و استادان کسی جواب سوال من رو میدونه؟؟؟؟؟؟؟ :sad: :puke: :whistle:
لطفا سریع جواب بدید.با تشکر

mofidy1
21-02-2007, 23:37
سلام به همه دوستان و استادان ریاضی
یه سوال امتیازی مطرح شده که 2 نمره داره ، هر کی زودتر بده ، نمره رو میگیره ، پس لطفا سریع کمک کنید.

طریقه حل ضرب خارجی در فضای 4 بعدی (یعنی به جای 3 متغیر ، 4 متغیر داشته باشه)
مثلا (a,b,c,d)*(x,y,z,g) ستاره به جای علامت ضرب خارجی به کار رفته.

لطفا روش حل کردن ضرب خارجی بالا رو به من بگید.(فقط زودتر ، تا 2 نمره بگیرم ، 2 نمره تو ریاضی 2 خیلیه;) )
با تشکر از همه دوستان

با سلام

(a,b,c,d)*(x,y,z,g) =(ax-by-cz-dg,ay+bx+cg-dz,az+xc+dy-bg,ag+dx+bz-cy)

موفق باشید.

ironroad
22-02-2007, 08:22
با استفاده از اصل لانه کبوتری اثبات کنید

به ازای هر عدد طبیعی حداقل یک مضرب وجود دارد که فقط از اعداد 0 و 1 تشکیل شده مثلا

عدد 4 مضرب 100 دارد یا عدد 2 مضرب های 10 و 100 و 110 و 1010 و ... دارد

eh_mn
22-02-2007, 12:44
سلام
فکر نمی کنم سوال ساده ای باشه ولی به نظر من خیلی جالبه:
فرض کنید P چند جمله ای با ضرایب صحیح باشدوضریب پیشرو ان یک باشد وd عددی طبیعی به طوریکه درجه P برd بخش پذیر باشد و معادله
[ برای مشاهده لینک ، لطفا با نام کاربری خود وارد شوید یا ثبت نام کنید ]
نا متناهی جواب برای m,n طبیعی داشته باشد.ثابت کنید چند جمله ای Q با ضرایب صحیح وجود دارد به طوری که برای هر x:
[ برای مشاهده لینک ، لطفا با نام کاربری خود وارد شوید یا ثبت نام کنید ]

با سلام

اگر ممکن است راهنمایی کنید!

mss_sdt
22-02-2007, 13:32
با سلام

(a,b,c,d)*(x,y,z,g) =(ax-by-cz-dg,ay+bx+cg-dz,az+xc+dy-bg,ag+dx+bz-cy)

موفق باشید.

دوست عزیز دستت درد نکنه. اگه میشه روش بدست آوردن این فرمول رو هم بزارید ، خیلی ممنون میشم;)
با تشکر از شما

ali_hp
22-02-2007, 14:06
سلام
فکر نمی کنم سوال ساده ای باشه ولی به نظر من خیلی جالبه:
فرض کنید P چند جمله ای با ضرایب صحیح باشدوضریب پیشرو ان یک باشد وd عددی طبیعی به طوریکه درجه P برd بخش پذیر باشد و معادله
[ برای مشاهده لینک ، لطفا با نام کاربری خود وارد شوید یا ثبت نام کنید ]
نا متناهی جواب برای m,n طبیعی داشته باشد.ثابت کنید چند جمله ای Q با ضرایب صحیح وجود دارد به طوری که برای هر x:
[ برای مشاهده لینک ، لطفا با نام کاربری خود وارد شوید یا ثبت نام کنید ]
سلام
فرض کنید درجه P برابر kd باشد.ثابت کنید چند جمله ای Q با ضرایب گویا و ضریب پیشرو مثبت و چند جمله ای R با ضرایب گویا و جود دارند به طوریکه :
[ برای مشاهده لینک ، لطفا با نام کاربری خود وارد شوید یا ثبت نام کنید ]
حالا ثابت کنید عددمثبت M موجود است به طوریکه برای هر x>M داشته باشیم:
[ برای مشاهده لینک ، لطفا با نام کاربری خود وارد شوید یا ثبت نام کنید ]
(تا اینجا نیازی به استفاده از اینکه معادله P(n)=m^d نا متناهی جواب دارد نبود.)
حالا با استفاده از اینکه معادله P(n)=m^d نا متناهی جواب دارد ثابت کنیدبرای هر X راریم:
[ برای مشاهده لینک ، لطفا با نام کاربری خود وارد شوید یا ثبت نام کنید ]
تعمیم این مساله که در پست 651 امده است هم تقریبا به روشی مشابه حل می شود.

ali_hp
22-02-2007, 15:30
با استفاده از اصل لانه کبوتری اثبات کنید
به ازای هر عدد طبیعی حداقل یک مضرب وجود دارد که فقط از اعداد 0 و 1 تشکیل شده مثلا
عدد 4 مضرب 100 دارد یا عدد 2 مضرب های 10 و 100 و 110 و 1010 و ... دارد
سلام
فرض کنید عدد طبیعی n داده شده است دقت کنید که با قیمانده تقسیم هر عدد بر n یکی از n عدد زیر است:
[ برای مشاهده لینک ، لطفا با نام کاربری خود وارد شوید یا ثبت نام کنید ]
n+1 چمله اول دنباله زیر را در نظر بگیرید:
[ برای مشاهده لینک ، لطفا با نام کاربری خود وارد شوید یا ثبت نام کنید ]
حالا چون n+1>n طبق اصل لانه کبوتری دو عدد از n+1 عدد در نظر گرفته شده باقیمانده یکسان در تقسیم بر n دارند.
به سادگی می توان دید که تفاضل این دو عدد شرایط مساله را دارد.

ali_hp
22-02-2007, 17:33
دنباله فیبوناتچی به صورت زیر تعریف می شود:
[ برای مشاهده لینک ، لطفا با نام کاربری خود وارد شوید یا ثبت نام کنید ]
و برای هر عدد طبیعی n :
[ برای مشاهده لینک ، لطفا با نام کاربری خود وارد شوید یا ثبت نام کنید ]
ثابت کنید برای هر عدد طبیعی m; نامتناهی از جملات دنباله فیبوناتچی بر m بخش پذیرند.

mss_sdt
23-02-2007, 20:16
از اساتید و دوستان کسی راه بدست آوردن فرمول
(a,b,c,d)*(x,y,z,g) =(ax-by-cz-dg,ay+bx+cg-dz,az+xc+dy-bg,ag+dx+bz-cy)
را نمیدونه؟
لطفا زودتر کمک کنید ، تا کسه دیگه ای جواب نداده و 2 نمره رو نگرفته:sad: ;)

mofidy1
24-02-2007, 14:03
با سلام

شکل یک چند جمله ای درجه ی 3 را که دارای یک ماکزیمم و یک می نیمم است، در نظر بگیرید. همه خطوطی را در نظر بگیرید که این شکل را در سه نقطه قطع می کنند. ثابت کنید مجموع طولهای این سه نقطه ثابت است و بستگی به خط ندارد. (راهنمایی: با مساوی قرار دادن فرمول خط و تابع درجه ی سه به یک معادله برسید و رابطه بین ریشه را در نظر بگیرید.)

آیا می توانید این مساله را تعمیم دهید؟

موفق باشید.

ارسال متن: شنبه 28 بهمن 1385

با سلام

برای حل، فرمول کلی منحنی درجه ی 3 و درجه ی1 را در نظر بگیرید:


[ برای مشاهده لینک ، لطفا با نام کاربری خود وارد شوید یا ثبت نام کنید ]

با حل دستگاه بالا و بنابر فرض مساله، معادله زیر دارای سه ریشه حقیقی است که همان طولهای نقاط تقاطع هستند:


[ برای مشاهده لینک ، لطفا با نام کاربری خود وارد شوید یا ثبت نام کنید ]

فرض کنید a_2،a_1 و a_3 این ریشه ها باشند. با توجه به روابط بین ریشه ها می توان نوشت:


[ برای مشاهده لینک ، لطفا با نام کاربری خود وارد شوید یا ثبت نام کنید ]

که حل مساله را کامل می کند.

با کمی دقت می توان مساله را به صورت زیر تعمیم داد:

فرض کنید P یک چند جمله ای از درجه n باشد که n حداقل2 است. چند جمله ایهایی از درجه ی حداکثر n-2 که P را دقیقاً در n نقطه قطع می کنند، در نظر بگیرید. ثابت کنید مجموع طولهای این n نقطه ثابت است و بستگی به چند جمله ایها ندارد.

موفق باشید.

ارسال متن: شنبه 5 اسفند 1385

attractive_girl
24-02-2007, 14:25
سلام
با استفاده از فرمول مواور و بسط دو جمله ای می توان روابط زیر را بدست اورد:
[ برای مشاهده لینک ، لطفا با نام کاربری خود وارد شوید یا ثبت نام کنید ]

ممنون میشه بیشتر توضیح بدین؟

attractive_girl
24-02-2007, 14:31
«في» (...Φ=1/618033988749895) عددي گنگ (Irrational) مانند:‌ عدد «پي» ( =...1459265358979/3) است و داراي ويژگي‌هاي رياضي غيرمعمول است و لي برخلاف عدد «پي» ( ) - كه قابل بيان با يك رابطه‌ي جبري نيست - با رابطه‌‌ي جبري از درجه‌ي دو قابل بيان است
نسبت طلايي
نسبت يا تناسب با ضريب عدد «في» (Φ) داراي ويژگي‌هايي است كه با بيان‌هاي ذيل تعريف شده است:

يونانيان باستان
«تقسيم يك خط به‌نسبت يا تناسب بي‌نهايت»

هنرمندان دوره‌ي رونسانس
«نسبت الهي»

نسبت، تناسب يا متوسط طلايي

محاسبه‌‌ها در سري اعداد
در قرن دوازدهم ميلادي، «لئوناردو فيبوناچي» (Leonardo Fibonacci) (شكل 4) يك‌سري عددي ساده‌اي را كشف كرد كه اساس رابطه‌اي باورنكردني رياضي است كه بيان‌گر عدد «في» (Φ) محسوب مي‌شود. اين سري با صفر و يك شروع مي‌شود و هر عدد در دنباله، از مجموع دو عدد قبلي حاصل مي‌شود:
...، 144، 89، 55، 34، 21، 13، 8، 2، 1، 1، 0

نسبت هر عدد بر عدد قبلي در دنباله‌ي كشف شده به‌عدد «في» (Φ) نزديك است. مثلا: حاصل تقسيم 5 بر 3 برابر است با: ...666/1 و نسبت 8 بر 5 عبارت است از: 60/1 و ...

m_honarmand_j
24-02-2007, 14:48
سلام
ثابت کنید که می توان چند دایره را در صفحه رسم کرد به طوری که نقطه ی برخورد درونی نداشته باشند و هر یک از آنها درست بر پنج دایره ی دیگر مماس باشد .

ali_hp
24-02-2007, 14:54
ممنون میشه بیشتر توضیح بدین؟
سلام
دو تا فرمولی که گفتم sin nx وcos nx را بر حسب sinx , cosx بیان می کنند.حالا اگر فرمولها مبهم است و با نماد سیگما مشکل دارید بگید تا فرمول ها رو توضیح بدم
اگراصلا این فرمولها مد نظرتون نبوده سوالتونو دقیقتر بگید.

m_honarmand_j
24-02-2007, 14:55
سلام
یک سوال جالب که من خودم نتونستم حلش کنم :
یک 2 به توان n ضلعی داریم که هر ضلع آن با دو رنگ قرمز یا آبی رنگ شده است . در هر مرحله هر ضلع را اگر دو ضلع مجاورش دارای رنگ های یکسان باشند به رنگ قرمز و اگر دارای رنگ های مختلف باشند به رنگ آبی در می آوریم . ثابت کنید که بعد از 2 به توان n-1 بار انجام دادن این عمل رنگ همه ی ضلع ها یکسان می شود .

m_honarmand_j
24-02-2007, 15:03
سلام
خط شکسته ی بسته ای که پنج ضلع دارد یک پنج ضلعی ستاره ای با زاویه های برابر ساخته است . اگر طول خط شکسته برابر واحد باشد ، محیط پنج ضلعی درونی چیست؟

mofidy1
24-02-2007, 20:16
با سلام

فرض کنید تابع حقیقی f بر بازه بسته ی [a,b] مشتق پذیر باشد و 0=(f(a)=f(b. ثابت کنید برای هر n از اعداد طبیعی، c_n در بازه ی باز (a,b) چنان موجود است که:


[ برای مشاهده لینک ، لطفا با نام کاربری خود وارد شوید یا ثبت نام کنید ]

موفق باشید.

ارسال متن: شنبه 5 اسفند 1385

mahshid_glna
24-02-2007, 21:02
سلام
من چند تا مسئله جالب و خوب در مورد اعداد ناشمارا لازم داشتم نتونستم خودم چیز زیادی پیدا کنم اگه کسی میتونه کمکم کنه ممنون میشم .:sad:

ali_hp
24-02-2007, 21:53
با سلام

فرض کنید تابع حقیقی f بر بازه بسته ی [a,b] مشتق پذیر باشد و (f(a)=f(b. ثابت کنید برای هر n از اعداد طبیعی، c_n در بازه ی باز (a,b) چنان موجود است که:


[ برای مشاهده لینک ، لطفا با نام کاربری خود وارد شوید یا ثبت نام کنید ]

موفق باشید.

ارسال متن: شنبه 5 اسفند 1385
سلام اقای مفیدی فکر می کنم سوال باید تصحیح بشه.
مثال نقض:اگر صفر در بازه [a,b] نباشد و f یک تابع ثابت و غیر صفر باشد برای هر xدر [a,b] مقدار x^n*f(x)l نا صفر است و لی مقدار اف پریم در سراسر بازه صفر است پس هیچگاه با هم برابر نمی شوند.

Hazy
24-02-2007, 22:04
ها!
اين مسئله هفت در هفت رو كي تونسته حل كنه!!!!!!!!!همنونه كي تقسيم و همش ستاره است

mofidy1
24-02-2007, 22:51
سلام اقای مفیدی فکر می کنم سوال باید تصحیح بشه.
مثال نقض:اگر صفر در بازه [a,b] نباشد و f یک تابع ثابت و غیر صفر باشد برای هر xدر [a,b] مقدار x^n*f(x)l نا صفر است و لی مقدار اف پریم در سراسر بازه صفر است پس هیچگاه با هم برابر نمی شوند.

با سلام

حق با شماست، صورت سوال تصحیح شد.

ممنون

ironroad
25-02-2007, 22:39
اگر f(x-1/x)=x^3+1/x^3 مقدارf(x را بیابید

ali_hp
26-02-2007, 13:10
سلام
من چند تا مسئله جالب و خوب در مورد اعداد ناشمارا لازم داشتم نتونستم خودم چیز زیادی پیدا کنم اگه کسی میتونه کمکم کنه ممنون میشم .:sad:
سلام
خوب اگه می گفتیدمسایل از کدوم قسمت این مبحث باشن بهتر بود.
1) ثابت کنید اجتماع شماراتا مجموعه شمارا مجموعه ای شمارا می شود.
2) ثابت کنید مجموعه اعداد حقیقی و مجموعه اعداد طبیعی هم ارز نیستند(به عبارت دیگر ثابت کنید مجموعه اعداد حقیقی ناشمارا است)
3)عدد حقیقی a را جبری می نامیم هرگاه یک چند جمله ای با ضرایب صحیح و درجه مثبت موجود باشد به طوریکه a ریشه ان چند جمله ای باشد.ایا همه اعداد حقیقی جبری هستند؟
4) نشان دهید خط و صفحه هم ارزند.(یعنی یک تابع یک یه یک و پوشا از نقاط خط به نقاط صفحه وجود دارد.)
این هم دو مساله متفاوت:
ایا تعداد مولکولهای موجود در جهان سه بعدی ما شما را است؟ تعداد اجسام چطور؟
ایا تعداد عبارات مختلف به زبا ن فارسی شما را است؟ایا بشر قادر است همه اعداد حقیقی را با عبارات زبان فارسی نامگذاری کند؟

m_honarmand_j
27-02-2007, 01:22
سلام
یک سوال جالب که من خودم نتونستم حلش کنم :
یک 2 به توان n ضلعی داریم که هر ضلع آن با دو رنگ قرمز یا آبی رنگ شده است . در هر مرحله هر ضلع را اگر دو ضلع مجاورش دارای رنگ های یکسان باشند به رنگ قرمز و اگر دارای رنگ های مختلف باشند به رنگ آبی در می آوریم . ثابت کنید که بعد از 2 به توان n-1 بار انجام دادن این عمل رنگ همه ی ضلع ها یکسان می شود .

سلام
می خواستم بگم که من این سوال و حل کردم ولی تو دفتر تقریبا 8 صفحه شد . اگه کسی خواست بگه تا براش جواب و بزارم .

mss_sdt
28-02-2007, 13:35
سلام به همه دوستان
لطفا به این سوال جواب بدید.خیلی ممنون میشم اگه یکم سریعتر باشه.
[ برای مشاهده لینک ، لطفا با نام کاربری خود وارد شوید یا ثبت نام کنید ]
استادان ریاضی کمک کنید تا این مساله حل بشه.دستتون درد نکنه

m_honarmand_j
01-03-2007, 18:46
سلام
فرض کنید (f(x و (g(x چند جمله ای های غیر صفر باشند که (f(x^2+x+1) = f(x)g(x نشان دهید (f(x درجه ی زوج است .
(دوستان به اتاق ترکیبیات هم سر بزنید)

mofidy1
02-03-2007, 19:43
با سلام

فرض کنید تابع حقیقی f بر بازه بسته ی [a,b] مشتق پذیر باشد و 0=(f(a)=f(b. ثابت کنید برای هر n از اعداد طبیعی، c_n در بازه ی باز (a,b) چنان موجود است که:


[ برای مشاهده لینک ، لطفا با نام کاربری خود وارد شوید یا ثبت نام کنید ]

موفق باشید.

ارسال متن: شنبه 5 اسفند 1385

با سلام

برای هر n از اعداد طبیعی قرار دهید:


[ برای مشاهده لینک ، لطفا با نام کاربری خود وارد شوید یا ثبت نام کنید ]

با استفاده از فرض مساله داریم: g(a)=g(b)=0. حال بنابر قضیه رول، c_n در بازه ی باز (a,b) چنان موجود است که:
g'(c_n)=0. بنابر این با محاسبه مشتق g حل مساله کامل می شود.

موفق باشید.

ارسال متن: جمعه 11 اسفند 1385

mofidy1
03-03-2007, 15:34
با سلام

در مثلث ABC زاویه B دو برابر زاویه C است. ثابت کنید که:


[ برای مشاهده لینک ، لطفا با نام کاربری خود وارد شوید یا ثبت نام کنید ]

موفق باشید.

ارسال متن: شنبه 12 اسفند 1385

abay
05-03-2007, 18:38
سلام
من دوتا سوال داشتم اگه دوستان بتونن زود جواب بدن ممنون میشم
1)در دنباله ی [ برای مشاهده لینک ، لطفا با نام کاربری خود وارد شوید یا ثبت نام کنید ]از جمله ی سوم به بعد هر جمله برابر است با جمله ی قبلی منهای جمله پیش از جمله ی قبلی . اگر[ برای مشاهده لینک ، لطفا با نام کاربری خود وارد شوید یا ثبت نام کنید ]آنگاه مجموع 245جمله ی اول این دنباله را بدست اورید.
2)اگر[ برای مشاهده لینک ، لطفا با نام کاربری خود وارد شوید یا ثبت نام کنید ]وباقیمانده یnبر4برابرصفر یا یک باشدحاصل عبارت مقابل را بدست آورید

[ برای مشاهده لینک ، لطفا با نام کاربری خود وارد شوید یا ثبت نام کنید ]

abay
05-03-2007, 19:34
[QUOTE=mofidy1;902262]با سلام

در مثلث ABC زاویه B دو برابر زاویه C است. ثابت کنید که:


[ برای مشاهده لینک ، لطفا با نام کاربری خود وارد شوید یا ثبت نام کنید ]

موفق باشید.



[ برای مشاهده لینک ، لطفا با نام کاربری خود وارد شوید یا ثبت نام کنید ][/
به مثلث[ برای مشاهده لینک ، لطفا با نام کاربری خود وارد شوید یا ثبت نام کنید ]مثلث[ برای مشاهده لینک ، لطفا با نام کاربری خود وارد شوید یا ثبت نام کنید ]اضافه میکنیم طوریکه[ برای مشاهده لینک ، لطفا با نام کاربری خود وارد شوید یا ثبت نام کنید ]که دراینصورت مثلث[ برای مشاهده لینک ، لطفا با نام کاربری خود وارد شوید یا ثبت نام کنید ]متساوی الساقین است واگر ارتفاع این مثلث یعنیAHرارسم کنیم داریم:[ برای مشاهده لینک ، لطفا با نام کاربری خود وارد شوید یا ثبت نام کنید ]وبا توجه به قضیه فیثاغورث در مثلثهای قائم الزاویه یAHC,ABHداریم:[ برای مشاهده لینک ، لطفا با نام کاربری خود وارد شوید یا ثبت نام کنید ]و همچنین داریم:[ برای مشاهده لینک ، لطفا با نام کاربری خود وارد شوید یا ثبت نام کنید ]وبا توجه به صورت مسئله که گفته زاویه ی BدوبرابرCاست خواهیم داشت:[ برای مشاهده لینک ، لطفا با نام کاربری خود وارد شوید یا ثبت نام کنید ]بنابراین مثلث اضافه شده نیز متساوی الساقین است ودر این مثلث اضلاع مقابل به زوایای مساوی برابرندیعنی:[ برای مشاهده لینک ، لطفا با نام کاربری خود وارد شوید یا ثبت نام کنید ]حال باتوجه به رابطه ی*میتوان نوشت:[ برای مشاهده لینک ، لطفا با نام کاربری خود وارد شوید یا ثبت نام کنید ]که نتیجه میدهد[ برای مشاهده لینک ، لطفا با نام کاربری خود وارد شوید یا ثبت نام کنید ]

ali_hp
05-03-2007, 22:14
سلام
من دوتا سوال داشتم اگه دوستان بتونن زود جواب بدن ممنون میشم
1)در دنباله ی [ برای مشاهده لینک ، لطفا با نام کاربری خود وارد شوید یا ثبت نام کنید ]از جمله ی سوم به بعد هر جمله برابر است با جمله ی قبلی منهای جمله پیش از جمله ی قبلی . اگر[ برای مشاهده لینک ، لطفا با نام کاربری خود وارد شوید یا ثبت نام کنید ]آنگاه مجموع 245جمله ی اول این دنباله را بدست اورید.
2)اگر[ برای مشاهده لینک ، لطفا با نام کاربری خود وارد شوید یا ثبت نام کنید ]وباقیمانده یnبر4برابرصفر یا یک باشدحاصل عبارت مقابل را بدست آورید

[ برای مشاهده لینک ، لطفا با نام کاربری خود وارد شوید یا ثبت نام کنید ]
سلام abay عزیز .
1)
داریم
[ برای مشاهده لینک ، لطفا با نام کاربری خود وارد شوید یا ثبت نام کنید ]
با جمع زدن روابط بالا بدست می اید:
[ برای مشاهده لینک ، لطفا با نام کاربری خود وارد شوید یا ثبت نام کنید ]
از طرفی داریم:
[ برای مشاهده لینک ، لطفا با نام کاربری خود وارد شوید یا ثبت نام کنید ]
با توجه به اینکه 244=6*40+4 داریم:
[ برای مشاهده لینک ، لطفا با نام کاربری خود وارد شوید یا ثبت نام کنید ]
2)
[ برای مشاهده لینک ، لطفا با نام کاربری خود وارد شوید یا ثبت نام کنید ]

mofidy1
09-03-2007, 18:23
با سلام

دوستان عزیز از گذاشتن تصویرهای پرحجم در پستهای خود خودداری کنید. این تصاویر در مطالعه و لود شدن صفحات به شدت مزاحمت ایجاد می کنند، مخصوصا با این سرعتهای لاک پشتی موجود در کشور. از abay در پست 686و mss_sdt در پست 681خواهش میکنم که مطالب خود را ویرایش کنند و به جای تصویر، یک لینک معرفی کنند.

متشکرم.

mofidy1
09-03-2007, 18:26
با سلام

در مثلث ABC زاویه B دو برابر زاویه C است. ثابت کنید که:


[ برای مشاهده لینک ، لطفا با نام کاربری خود وارد شوید یا ثبت نام کنید ]

موفق باشید.

ارسال متن: شنبه 12 اسفند 1385

با سلام

از abay که در پست 686 ([ برای مشاهده لینک ، لطفا با نام کاربری خود وارد شوید یا ثبت نام کنید ]) به حل مساله پرداختند، تشکر می کنم. برای دیدن راه حل ایشان به لینک زیر مراجعه فرمایید:

[ برای مشاهده لینک ، لطفا با نام کاربری خود وارد شوید یا ثبت نام کنید ]

بنده راه حل دیگری را خدمتتان تقدیم می کنم:

نیمساز زاویه B را رسم کنید تا AC را در نقطه D قطع کند. دو مثلث ABD و ABC مشابهند؛ لذا:


[ برای مشاهده لینک ، لطفا با نام کاربری خود وارد شوید یا ثبت نام کنید ]

این مساله در مسابقه طراحی سوالات خلاق برای معلمین استان فارس مطرح شده بود.

موفق باشید.

ارسال متن: جمعه 18 اسفند 1385

mofidy1
10-03-2007, 11:53
با سلام

A را مجموعه اعداد طبیعی 1 تا 1000 فرض کنید. اگر n عضوی از A باشد، ثابت کنید که لگاریتم n در پایه 10عددی گویاست اگر و فقط اگر n یکی از اعداد 1، 10، 100 و 1000 باشد.

موفق باشید.

ارسال متن: شنبه 19 اسفند 1385 ( مطابق با اربعین حسینی (ع) )

abay
10-03-2007, 16:10
سلام abay عزیز .
1) جواب این سوال بر حسب جمله اول و دوم بدست میاد.
داریم
[ برای مشاهده لینک ، لطفا با نام کاربری خود وارد شوید یا ثبت نام کنید ]
با جمع زدن روابط بالا بدست می اید:
[ برای مشاهده لینک ، لطفا با نام کاربری خود وارد شوید یا ثبت نام کنید ]
از طرفی داریم:
[ برای مشاهده لینک ، لطفا با نام کاربری خود وارد شوید یا ثبت نام کنید ]
با توجه به اینکه 244=6*40+4 داریم:
[ برای مشاهده لینک ، لطفا با نام کاربری خود وارد شوید یا ثبت نام کنید ]
2)

[ برای مشاهده لینک ، لطفا با نام کاربری خود وارد شوید یا ثبت نام کنید ]
سلام
ali-hpجان خیلی ممنون از پاسخگوییتان
فقط مسئله ای که است جواب سوال یک میباشد که من در جاییکه سوال را دیده ام جواب آن عدد 2است لطفا" یه توضیحی در این مورد بدهید

ali_hp
10-03-2007, 23:59
سلام
ali-hpجان خیلی ممنون از پاسخگوییتان
فقط مسئله ای که است جواب سوال یک میباشد که من در جاییکه سوال را دیده ام جواب آن عدد 2است لطفا" یه توضیحی در این مورد بدهید
سلام
من در حل مساله یک فرض کرده بودم برای هر n "بزرگتر از سه" جمله nام برابر است با جمله n-1 به اضافه جمله n-2 .ولی مثل اینکه منظور سوال اینه که برای هر n "بزرگتر یا مساوی سه"
جمله nام برابر است با جمله n-1 به اضافه جمله n-2 .که در این صورت می توان جمله اول و دوم را محاسبه کرد و جواب مساله را بدست اورد:
[ برای مشاهده لینک ، لطفا با نام کاربری خود وارد شوید یا ثبت نام کنید ]
در پست 687 راه حل مساله رو ویرایش کردم که کمی هم ساده تر شده است

mamoosh0007
12-03-2007, 11:25
مجموعه (1و1) چند عضو دارد

ali1234
13-03-2007, 09:44
اگر منظورتان {1 و 1} است .كه يك عظو دارد.

mofidy1
17-03-2007, 21:21
با سلام

A را مجموعه اعداد طبیعی 1 تا 1000 فرض کنید. اگر n عضوی از A باشد، ثابت کنید که لگاریتم n در پایه 10عددی گویاست اگر و فقط اگر n یکی از اعداد 1، 10، 100 و 1000 باشد.

موفق باشید.

ارسال متن: شنبه 19 اسفند 1385 ( مطابق با اربعین حسینی (ع) )

حل مساله:

مجموعه A را به سه قسمت افراز کنید:

الف) اعداد 1، 10، 100 و 1000؛

ب) اعدادی که در تجزیه آنها فقط 2 یا 5 یا هر دو ظاهر می شود اما 10، 100 و 1000 نیستند،

ج) اعدادی که بر یک عدد اول فرد غیر از 5 بخشپذیرند.

با کمی دقت و با کمک برهان خلف می توان ثابت کرد که لگاریتم اعداد قسمت (ب) و قسمت (ج) ، گویا نیستند که با این کار، حل مساله کامل می شود.

موفق باشید.

ارسال متن: شنبه 27 اسفند 1385

mofidy1
17-03-2007, 21:48
با سلام

سکه ای را n بار پرتاب می کنیم. ثابت کنید که اگر n>2 ، احتمال آنکه در بین پرتابها دوبار متوالی پشت بیاید، برابر است با


[ برای مشاهده لینک ، لطفا با نام کاربری خود وارد شوید یا ثبت نام کنید ]

که F_n جمله ی n-ام دنباله فیبوناتچی است.

موفق باشید.

سال نو بر همه دوستان عزیز، مبارک و میمون. در پناه حق موفق و موید و منصور باشید. دعا کنید امسال، سالی خوب، پربرکت و همراه با موفقیت برای کشور عزیزمان در همه عرصه های مادی و معنوی باشد. انشاءالله.

اللهم عجل لولیک الفرج و النصر و العافیة و صل علی محمد و آله الطاهرین.

ارسال متن: شنبه 26 اسفند 1385

M A T I N
19-03-2007, 16:55
نمی دونم باید سوالم رو اینجا بپرسم یا ....
یکی از سوالات المپیاد ریاضی این بود :
================================================
چه تعداد از پاره خطهای بین نقاط زیر محور x ها را قطع میکند ؟
[ برای مشاهده لینک ، لطفا با نام کاربری خود وارد شوید یا ثبت نام کنید ]

الف) 4
ب ) 6
ج ) 16
د) 21
ه ) 24

================================================
خودم اینجوری حلش کردم :
چون چهار تا از نقطه ها y شون منفی هست پس پایین محور x هاست
6 تاشون هم y هاشون مثبته پس بالای محور x هاست ....
24=4*6
من خودم ه رو زدم ....
ولی دیروز که کلید سوالاتشو از سایتش گرفتم زده بود 6 بار ( گزینه ب ) محور x ها رو قطع مکنه ....

حالا کسی میدونه اشتباه من کجاست ...؟

.........خیلی ممنون .........

ali_hp
19-03-2007, 19:25
نمی دونم باید سوالم رو اینجا بپرسم یا ....
یکی از سوالات المپیاد ریاضی این بود :
================================================
چه تعداد از پاره خطهای بین نقاط زیر محور x ها را قطع میکند ؟
[ برای مشاهده لینک ، لطفا با نام کاربری خود وارد شوید یا ثبت نام کنید ]

الف) 4
ب ) 6
ج ) 16
د) 21
ه ) 24

================================================
خودم اینجوری حلش کردم :
چون چهار تا از نقطه ها y شون منفی هست پس پایین محور x هاست
6 تاشون هم y هاشون مثبته پس بالای محور x هاست ....
24=4*6
من خودم ه رو زدم ....
ولی دیروز که کلید سوالاتشو از سایتش گرفتم زده بود 6 بار ( گزینه ب ) محور x ها رو قطع مکنه ....

حالا کسی میدونه اشتباه من کجاست ...؟

.........خیلی ممنون .........
سلام
به نظر من راه حلتون کاملادرسته.
شاید کلید مربوط به یک کد دیگرو دیدین شاید هم کلید اشتباه باشه.
===============================================

امیدوارم همه عزیزان سال خوبی داشته باشند.

M A T I N
19-03-2007, 19:40
سلام
به نظر من راه حلتون کاملادرسته.
شاید کلید مربوط به یک کد دیگرو دیدین شاید هم کلید اشتباه باشه.

سلام
کد سوالات درسته .... جواب بقیه سوال هایی که حل کردم هم درسته ...
من هم شک کردم که شاید جواب این سوالش اشتباهه .... چون این سوال به نظر خودم آسون ترین سوالش بود...
خیلی ممنون از جوابتون ...

AMIRJONJON
20-03-2007, 01:54
سوال: اگر S مساحت مثلث نامشخص ABC وS1 مساحت مثلثی به اضلاع
1/ha و
1/hb و
1/hc
باشد ثابت کنید S . S1 =1/4
توجه : این مسله کاملا تازگی دارد و تا کنون در هیچ جایی دیده نشده است .
منظور از 1/ha یک تقسیم بر ارتفاع نظیر راس A است که به علت کمبود امکانات به این صورت نوشته شده است. در عبارتی که قرار است ثابت کنید منظور از نقطه علامت ضرب است .

msm43njn
20-03-2007, 18:52
من کاربر تازه کار این قسمت فوروم هستم.
من اشکالاتم را چطور باید بپرسم در صورتی که هیچ امکانی برای وارد کردن کاراکترهای ویژه ریاضی در اینجا وجود ندارد؟

M A T I N
21-03-2007, 12:55
من کاربر تازه کار این قسمت فوروم هستم.
من اشکالاتم را چطور باید بپرسم در صورتی که هیچ امکانی برای وارد کردن کاراکترهای ویژه ریاضی در اینجا وجود ندارد؟


فکر می کنم لازم است در آینده روش مناسب تایپ ریاضی در صفحات وب را به بحث بگذاریم. برای مثال به دو صفحه زیر در همین رابطه مراجعه فرمایید:

[ برای مشاهده لینک ، لطفا با نام کاربری خود وارد شوید یا ثبت نام کنید ]

[ برای مشاهده لینک ، لطفا با نام کاربری خود وارد شوید یا ثبت نام کنید ] ([ برای مشاهده لینک ، لطفا با نام کاربری خود وارد شوید یا ثبت نام کنید ]) __________________

.................................................. ........

mofidy1
21-03-2007, 14:18
من کاربر تازه کار این قسمت فوروم هستم.
من اشکالاتم را چطور باید بپرسم در صورتی که هیچ امکانی برای وارد کردن کاراکترهای ویژه ریاضی در اینجا وجود ندارد؟

با سلام

دوست عزیز در تاپیک قوانین و فهرست مطالب انجمن ریاضیات ([ برای مشاهده لینک ، لطفا با نام کاربری خود وارد شوید یا ثبت نام کنید ]) در بخش مباحثی پیرامون زبان TEX و چگونگی انتشار فرمولهای ریاضی و تصاویر هندسی در اینترنت توضیحات کافی در این زمینه داده شده است. مثلا به لینک زیر مراجعه فرمایید:

[ برای مشاهده لینک ، لطفا با نام کاربری خود وارد شوید یا ثبت نام کنید ]

باز هم اگر مشکلی پیش آمد، مطرح فرمایید.

موفق باشید.

اول فروردین 1386

mofidy1
24-03-2007, 11:32
با سلام

سکه ای را n بار پرتاب می کنیم. ثابت کنید که اگر n>2 ، احتمال آنکه در بین پرتابها دوبار متوالی پشت بیاید، برابر است با


[ برای مشاهده لینک ، لطفا با نام کاربری خود وارد شوید یا ثبت نام کنید ]

که F_n جمله ی n-ام دنباله فیبوناتچی است.

موفق باشید.

سال نو بر همه دوستان عزیز، مبارک و میمون. در پناه حق موفق و موید و منصور باشید. دعا کنید امسال، سالی خوب، پربرکت و همراه با موفقیت برای کشور عزیزمان در همه عرصه های مادی و معنوی باشد. انشاءالله.

اللهم عجل لولیک الفرج و النصر و العافیة و صل علی محمد و آله الطاهرین.

ارسال متن: شنبه 26 اسفند 1385

با سلام

فرض کنید احتمال آنکه در n پرتاب، هیچگاه دو پشت متوالی ظاهر نشود، P_n باشد. لذا P_1 و P_2 به ترتیب یک و سه چهارم است. حال فرض کنید n>2. اگر اولین پرتاب رو بیاید، واضح است که احتمال تعریف شده {P_{n-1 است. اگر اولین پرتاب پشت بیاید، باید دومین پرتاب رو باشد و لذا احتمال اینکه در n-2 پرتاب بعدی دو پشت متوالی رخ ندهد برابر است با {P_{n-2 . قرار دهید:


[ برای مشاهده لینک ، لطفا با نام کاربری خود وارد شوید یا ثبت نام کنید ]

لذا


[ برای مشاهده لینک ، لطفا با نام کاربری خود وارد شوید یا ثبت نام کنید ]

تساوی آخر همان دنباله فیبوناتچی است که {S_n=F_{n+2. بنابر این احتمال مورد نظر برابر است با


[ برای مشاهده لینک ، لطفا با نام کاربری خود وارد شوید یا ثبت نام کنید ]

موفق باشید.

ارسال متن: شنبه 4 فروردین 1386

mofidy1
24-03-2007, 11:53
با سلام

ثابت کنید تنها دو تابع پیوسته در رابطه زیر صدق می کنند: تابع صفر و تابع همانی؛


[ برای مشاهده لینک ، لطفا با نام کاربری خود وارد شوید یا ثبت نام کنید ]

موفق باشید.

ارسال متن: شنبه 4 فروردین 1386

SuB
24-03-2007, 13:10
نمی دونم باید سوالم رو اینجا بپرسم یا ....
یکی از سوالات المپیاد ریاضی این بود :
================================================
چه تعداد از پاره خطهای بین نقاط زیر محور x ها را قطع میکند ؟
[ برای مشاهده لینک ، لطفا با نام کاربری خود وارد شوید یا ثبت نام کنید ]

الف) 4
ب ) 6
ج ) 16
د) 21
ه ) 24

================================================
خودم اینجوری حلش کردم :
چون چهار تا از نقطه ها y شون منفی هست پس پایین محور x هاست
6 تاشون هم y هاشون مثبته پس بالای محور x هاست ....
24=4*6
من خودم ه رو زدم ....
ولی دیروز که کلید سوالاتشو از سایتش گرفتم زده بود 6 بار ( گزینه ب ) محور x ها رو قطع مکنه ....

حالا کسی میدونه اشتباه من کجاست ...؟

.........خیلی ممنون .........

اشتباه شما اینه که سوال رو کامل ننوشتی. سوال رو کامل بنویس (حتی با کاما و نقطه هاش).
اگه سوال بگه چند تا از پاره خطها محور x ها رو قطع می کنند، جواب یه چیزه و اگه سوال این باشه که محور x ها رو کلاً در چند نقطه قطع می کنند، جواب یه چیزه دیگه است.

abay
24-03-2007, 21:25
با سلام

ثابت کنید تنها دو تابع پیوسته در رابطه زیر صدق می کنند: تابع صفر و تابع همانی؛


[ برای مشاهده لینک ، لطفا با نام کاربری خود وارد شوید یا ثبت نام کنید ]

موفق باشید.

ارسال متن: شنبه 4 فروردین 1386

[ برای مشاهده لینک ، لطفا با نام کاربری خود وارد شوید یا ثبت نام کنید ]
[ برای مشاهده لینک ، لطفا با نام کاربری خود وارد شوید یا ثبت نام کنید ]
پس داریم:[ برای مشاهده لینک ، لطفا با نام کاربری خود وارد شوید یا ثبت نام کنید ]که درواقع حالتی ازقضیه ی کوشی است

[ برای مشاهده لینک ، لطفا با نام کاربری خود وارد شوید یا ثبت نام کنید ]
وبه استقرا به ازای هر nثابت میشودکه:[ برای مشاهده لینک ، لطفا با نام کاربری خود وارد شوید یا ثبت نام کنید ]که این رابطه برایt<0 نیز برقرار است وبا توجه به قضیه کوشی داریم:f(x)=axواز انجا خواهیم داشت:
[ برای مشاهده لینک ، لطفا با نام کاربری خود وارد شوید یا ثبت نام کنید ]

abay
24-03-2007, 21:41
با سلام خدمت آقای مفیدی
من ازشما درخواستی داشتم واون اینکه اگه لینکی برای دانلود نرم افزار metapostسراغ داشتید قرار دهید.
با تشکر

mofidy1
25-03-2007, 00:01
با سلام خدمت آقای مفیدی
من ازشما درخواستی داشتم واون اینکه اگه لینکی برای دانلود نرم افزار metapost سراغ داشتید قرار دهید.
با تشکر

با سلام

دوست عزیز metapost نرم افزار نیست و یکی از قابلیتهای مفید زبان LATEX است. معمولا به وسیله نرم افزار Winedt و با استفاده از دستورات metapost می توان تصاویر بسیار زیبایی را ایجاد و به متن مقالات و کتب اضافه کرد. البته کار با آن تبحر خاصی می خواهد و فراگیری آن خیلی راحت نیست.

موفق باشید.

4 فروردین 1386

Mahdi_Co_1368
25-03-2007, 01:07
آقای mofidy1 اگه زحمتی نیست خودتون سوالاتی را برای مقطع سوم ریاضی گرداوری کنید و به صورت فایل pDF برای مشتاقان این درس بگذارید..
با تشکر از شما

M A T I N
25-03-2007, 14:35
اشتباه شما اینه که سوال رو کامل ننوشتی. سوال رو کامل بنویس (حتی با کاما و نقطه هاش).
اگه سوال بگه چند تا از پاره خطها محور x ها رو قطع می کنند، جواب یه چیزه و اگه سوال این باشه که محور x ها رو کلاً در چند نقطه قطع می کنند، جواب یه چیزه دیگه است.

الان دوباره نگاه کردم .... سوال دقیقا همینه ....

شاید هم به خاطر این غلط باشه که بعضی از نقاطی که محور x ها رو قطع میکنه روی هم منطبق بشن ...

SuB
26-03-2007, 12:17
شما سوال رو كامل بنويس تا بهت بگم.

ali_hp
27-03-2007, 13:57
سوال: اگر S مساحت مثلث نامشخص ABC وS1 مساحت مثلثی به اضلاع
1/ha و
1/hb و
1/hc
باشد ثابت کنید S . S1 =1/4
توجه : این مسله کاملا تازگی دارد و تا کنون در هیچ جایی دیده نشده است .
منظور از 1/ha یک تقسیم بر ارتفاع نظیر راس A است که به علت کمبود امکانات به این صورت نوشته شده است. در عبارتی که قرار است ثابت کنید منظور از نقطه علامت ضرب است .
سلام
داریم:
[ برای مشاهده لینک ، لطفا با نام کاربری خود وارد شوید یا ثبت نام کنید ]
پس مثلث ABC با مثلثی که اضلاع ان معکوسهای ارتفاعات مثلث ABC هستند متشابه است.با نسبت تشابه 2S
پس نسبت مساحتهای انها مجذور نسبت تشابه است و داریم:
[ برای مشاهده لینک ، لطفا با نام کاربری خود وارد شوید یا ثبت نام کنید ]

mathlove
27-03-2007, 14:25
دوستان خوبم، به حل مساله ای که در پست 1 آورده ام ، می پردازیم. همانطور که در آنجا آورده ام سعی می کنیم روش حل، فقط در سطح حسابان سال سوم ریاضی و در حد امکان ساده تر باشد. این حقیر، روش ذکر شده در کتاب را به دلایلی مناسب دانش آموزان نمی دانم و به تجربه، حل زیر را برای تدریس مناسب تر می دانم:

تعریف کنید [ برای مشاهده لینک ، لطفا با نام کاربری خود وارد شوید یا ثبت نام کنید ]

لذا [ برای مشاهده لینک ، لطفا با نام کاربری خود وارد شوید یا ثبت نام کنید ] و در نتیجه تابع f صعودی است.
فرض کنید x>0. پس [ برای مشاهده لینک ، لطفا با نام کاربری خود وارد شوید یا ثبت نام کنید ] و لذا [ برای مشاهده لینک ، لطفا با نام کاربری خود وارد شوید یا ثبت نام کنید ]

به همین ترتیب می توان ثابت کرد که [ برای مشاهده لینک ، لطفا با نام کاربری خود وارد شوید یا ثبت نام کنید ]

حال تعریف کنید [ برای مشاهده لینک ، لطفا با نام کاربری خود وارد شوید یا ثبت نام کنید ]

با نکته ای که قبل از تعریف g گفته شد و به روش بالا ثابت کنید g صعودی است و لذا برای x های نامنفی می توان نوشت [ برای مشاهده لینک ، لطفا با نام کاربری خود وارد شوید یا ثبت نام کنید ] و در نتیجه [ برای مشاهده لینک ، لطفا با نام کاربری خود وارد شوید یا ثبت نام کنید ] و با فرض x>0 به دست می آوریم[ برای مشاهده لینک ، لطفا با نام کاربری خود وارد شوید یا ثبت نام کنید ] پس [ برای مشاهده لینک ، لطفا با نام کاربری خود وارد شوید یا ثبت نام کنید ]
حال با تغییر متغیر x=-u که x<0 و با استفاده از نتیجه بالا می توان گفت: [ برای مشاهده لینک ، لطفا با نام کاربری خود وارد شوید یا ثبت نام کنید ] که اثبات را کامل می کند.
---------------------------------------------------
منتظر نظرات و روشهای دیگر شما هستم.dsafafa
تابعهای این سوال دیده نمی شوند اگه بفرستید ممنون می شوم mathlove

mathlove
27-03-2007, 14:28
هنوز تابع ای سوال مشخص نیستند

lopez
28-03-2007, 19:34
سلام دوستان
من اثبات اين مسئله رو مي خوام:
تعداد مربع هاي ، صفحه شطرنج چند تا است؟

لطفا اثبات رو برام بذارين:46:

ممنون

abay
28-03-2007, 20:20
سلام
اگه دوستان بتونن این دو مسئله را حل کنند ممنون میشم.
1) 6 نفر در یک مهمانی حضور دارند احتمال اینکه حداقل دو نفر در یک روز هفته متولد شده با شند چیست؟
2)مجموعه های[ برای مشاهده لینک ، لطفا با نام کاربری خود وارد شوید یا ثبت نام کنید ] از اعداد طبیعی مفروضند اگر[ برای مشاهده لینک ، لطفا با نام کاربری خود وارد شوید یا ثبت نام کنید ]مجموع عضوهای[ برای مشاهده لینک ، لطفا با نام کاربری خود وارد شوید یا ثبت نام کنید ]باشد مجموع 20 جمله ی اول را بدست آورید.(n تعداد عضوهای هر مجموعه است.)

mofidy1
30-03-2007, 14:17
با سلام

ثابت کنید تنها دو تابع پیوسته در رابطه زیر صدق می کنند: تابع صفر و تابع همانی؛


[ برای مشاهده لینک ، لطفا با نام کاربری خود وارد شوید یا ثبت نام کنید ]

موفق باشید.

ارسال متن: شنبه 4 فروردین 1386

با سلام

از abay که در پست 707 ([ برای مشاهده لینک ، لطفا با نام کاربری خود وارد شوید یا ثبت نام کنید ]) مساله را حل کردند، تشکر می کنم. برای دیدن راه حل ایشان (البته با کمی تغییر) به لینک زیر مراجعه فرمایید:

[ برای مشاهده لینک ، لطفا با نام کاربری خود وارد شوید یا ثبت نام کنید ]

موفق باشید.

ارسال متن: جمعه 10 فروردین 1386

SuB
30-03-2007, 16:24
سلام
اگه دوستان بتونن این دو مسئله را حل کنند ممنون میشم.
1) 6 نفر در یک مهمانی حضور دارند احتمال اینکه حداقل دو نفر در یک روز هفته متولد شده با شند چیست؟
2)مجموعه های[ برای مشاهده لینک ، لطفا با نام کاربری خود وارد شوید یا ثبت نام کنید ] از اعداد طبیعی مفروضند اگر[ برای مشاهده لینک ، لطفا با نام کاربری خود وارد شوید یا ثبت نام کنید ]مجموع عضوهای[ برای مشاهده لینک ، لطفا با نام کاربری خود وارد شوید یا ثبت نام کنید ]باشد مجموع 20 جمله ی اول را بدست آورید.(n تعداد عضوهای هر مجموعه است.)

سوال دوم شما واضح نیست.

mofidy1
30-03-2007, 20:06
سلام دوستان
من اثبات اين مسئله رو مي خوام:
تعداد مربع هاي ، صفحه شطرنج چند تا است؟

لطفا اثبات رو برام بذارين:46:

ممنون


سلام
1) 6 نفر در یک مهمانی حضور دارند احتمال اینکه حداقل دو نفر در یک روز هفته متولد شده با شند چیست؟


با سلام

دوستان عزیز لطفاً این سوالات را در اتاق ترکیبیات ([ برای مشاهده لینک ، لطفا با نام کاربری خود وارد شوید یا ثبت نام کنید ]) مطرح فرمایید.

با تشکر

mofidy1
31-03-2007, 18:41
با سلام

فرض کنید b ،a و c اعداد حقیقی نامنفی باشند که کمتر یا مساوی 1 نیز هستند. ثابت کنید:


[ برای مشاهده لینک ، لطفا با نام کاربری خود وارد شوید یا ثبت نام کنید ]

موفق باشید.

ارسال متن: شنبه 11 فروردین 1386

Ar@m
04-04-2007, 11:13
[ برای مشاهده لینک ، لطفا با نام کاربری خود وارد شوید یا ثبت نام کنید ]

webgardi
06-04-2007, 18:21
سلام دوستان.
من سه تا معاده دیفرانسیل دارم که به نظرم دو تاش سخته تقریبا. دنبال حل دوتاش هستم.
براش هم حاضرم پول خرج کنم.
کسی می تونه کمک کنه؟

ببخشید اگه جای ناجوری این موضوع رو مطرح کردم ولی به نظرم اینجا جاش بود.

mofidy1
07-04-2007, 18:15
با سلام

فرض کنید b ،a و c اعداد حقیقی نامنفی باشند که کمتر یا مساوی 1 نیز هستند. ثابت کنید:


[ برای مشاهده لینک ، لطفا با نام کاربری خود وارد شوید یا ثبت نام کنید ]

موفق باشید.

ارسال متن: شنبه 11 فروردین 1386

با سلام

از silentcloud که در پست 722 مساله را حل کردند، تشکر می کنم. توجه کنید که اگر c=1 ، نامساوی واضح است. حال فرض کنید عدد c مخالف 1 باشد. ادامه راه حل را در لینک زیر مطالعه فرمایید که آنرا silentcloud ارسال کرده اند:

[ برای مشاهده لینک ، لطفا با نام کاربری خود وارد شوید یا ثبت نام کنید ]

موفق باشید.

ارسال متن: شنبه 18 فروردین 1386

ali_hp
07-04-2007, 23:40
سلام
اگه دوستان بتونن این دو مسئله را حل کنند ممنون میشم.
2)مجموعه های[ برای مشاهده لینک ، لطفا با نام کاربری خود وارد شوید یا ثبت نام کنید ] از اعداد طبیعی مفروضند اگر[ برای مشاهده لینک ، لطفا با نام کاربری خود وارد شوید یا ثبت نام کنید ]مجموع عضوهای[ برای مشاهده لینک ، لطفا با نام کاربری خود وارد شوید یا ثبت نام کنید ]باشد مجموع 20 جمله ی اول را بدست آورید.(n تعداد عضوهای هر مجموعه است.)
سلام
با کمی دقت می توان دید که اخرین عضو N_20 برابر است 210 زیرا:
[ برای مشاهده لینک ، لطفا با نام کاربری خود وارد شوید یا ثبت نام کنید ]
و باز هم با کمی دقت می توان دید که مجموع 20 جمله اول دنباله برابر است با مجموع اعداد طبیعی از یک تا اخرین عضو N_20 پس مجموع 20 جمله اول دنباله برابر است با:
[ برای مشاهده لینک ، لطفا با نام کاربری خود وارد شوید یا ثبت نام کنید ]

mofidy1
08-04-2007, 07:06
با سلام

مقدار کسینوس 36 درجه را حساب کنید (تمام جزئیات روش خود را توضیح دهید).

موفق باشید.

ارسال متن: یکشنبه 19 فروردین 1386

abay
09-04-2007, 18:02
سلام
با کمی دقت می توان دید که اخرین عضو N_20 برابر است 210 زیرا:
[ برای مشاهده لینک ، لطفا با نام کاربری خود وارد شوید یا ثبت نام کنید ]
و باز هم با کمی دقت می توان دید که مجموع 20 جمله اول دنباله برابر است با مجموع اعداد طبیعی از یک تا اخرین عضو N_20 پس مجموع 20 جمله اول دنباله برابر است با:
[ برای مشاهده لینک ، لطفا با نام کاربری خود وارد شوید یا ثبت نام کنید ]

سلام
ali-hpعزیز اگه میشه یه توضیحی در مورد بدست اوردن آخرین عضوN_20بدهید در ضمن من روی سوال غلط نوشتم مسئله مقدار S_20را خواسته و جواب سوال 4010است

ali_hp
10-04-2007, 00:08
سلام
ali-hpعزیز اگه میشه یه توضیحی در مورد بدست اوردن آخرین عضوN_20بدهید در ضمن من روی سوال غلط نوشتم مسئله مقدار S_20را خواسته و جواب سوال 4010است
برای درک بهتر ساختار N_i ها دقت کنید که:
اولین عدد طبیعی در N_1 قرار دارد و دوعدد طبیعی بعد ی در N_2 و سه عدد طبیعی بعد در N_3 و سه عدد طبیعی
بعدی در N_4 .
پس 1+2+3+4=10 عدد طبیعی ابتدایی در مجموعه های N_1وN_2وN_3 وN_4 قرار دارند و اعضای N_5 عبارتند از 5 عدد طبیعی بعدی، یعنی:11و12و13و14و15
با استدلال مشابه نتیجه می شود:
[ برای مشاهده لینک ، لطفا با نام کاربری خود وارد شوید یا ثبت نام کنید ]
و به عبارتی دیگر:
[ برای مشاهده لینک ، لطفا با نام کاربری خود وارد شوید یا ثبت نام کنید ]

mofidy1
13-04-2007, 10:54
با سلام

مقدار کسینوس 36 درجه را حساب کنید (تمام جزئیات روش خود را توضیح دهید).

موفق باشید.

ارسال متن: یکشنبه 19 فروردین 1386

با سلام

برای سادگی فرض کنید (x=cos(36. بنابراین x مثبت است. حال می توان نوشت:


[ برای مشاهده لینک ، لطفا با نام کاربری خود وارد شوید یا ثبت نام کنید ]

موفق باشید.

ارسال متن: جمعه 24 فروردین 1386

mofidy1
14-04-2007, 16:29
با سلام

حد زیر را بیابید:


[ برای مشاهده لینک ، لطفا با نام کاربری خود وارد شوید یا ثبت نام کنید ]

نظرتان درباره مقدار حد زیر که حالت کلی تر حد بالاست، چیست؟



[ برای مشاهده لینک ، لطفا با نام کاربری خود وارد شوید یا ثبت نام کنید ]

موفق باشید.

ارسال متن: شنبه 25 فروردین 1386

behnam karami
18-04-2007, 16:46
ببخشید یه اشتباه لپی بود[ برای مشاهده لینک ، لطفا با نام کاربری خود وارد شوید یا ثبت نام کنید ]

sohreh
18-04-2007, 20:17
سلام.من نمی خوام جواب کسی رو بدم...منتهی چون عضو جدیدم نمی دونم کجا سوال بپرسم...سوالم اینه که
1=!0 قرار داده یا اثبات داره؟
ممنون

sohreh
18-04-2007, 20:18
سلام.من نمی خوام جواب کسی رو بدم...منتهی چون عضو جدیدم نمی دونم کجا سوال بپرسم...سوالم اینه که
1=!0 قرار داده یا اثبات داره؟اگه داره چطوری اثبات می شه؟
ممنون

behnam karami
18-04-2007, 22:36
با سلام و عرض تبریک به خاطر ورود شما به این انجمن.
تا اونجایی که من اطلاع دارم باید قرارداد باشه

qaderi
19-04-2007, 18:09
سلام به تمام دوستان

یه سوال دارم اگر برام حلش کنید ممنون میشم

Y=|sin3x|+cos(tavane 2) 3x

ر ضمن کسینوس توان 2 هست

aminkarami
20-04-2007, 09:31
با سلام

حد زیر را بیابید:


[ برای مشاهده لینک ، لطفا با نام کاربری خود وارد شوید یا ثبت نام کنید ]

نظرتان درباره مقدار حد زیر که حالت کلی تر حد بالاست، چیست؟



[ برای مشاهده لینک ، لطفا با نام کاربری خود وارد شوید یا ثبت نام کنید ]

موفق باشید.

ارسال متن: شنبه 25 فروردین 1386

من این مساله را فکر می کنم حل کردم.
[ برای مشاهده لینک ، لطفا با نام کاربری خود وارد شوید یا ثبت نام کنید ]
مرسی امین

پاکر
21-04-2007, 02:02
البته نمیدونم مفید هست یا نه ولی آقای مفیدی من راه حل حد سینوسی رو که گفتین
می دونم.وقعا جالب که اهمیت میدین!

m_honarmand_j
22-04-2007, 19:20
سلام
همه ی چند جمله ای های با ضرایب حقیقی (P(x,y را بیابید که برای هر x و y داشته باشیم :
(P(x+y,x-y) = 2P(x,y

(دوستان به اتاق ترکیبیات هر سر بزنید)

SuB
23-04-2007, 15:48
سلام.من نمی خوام جواب کسی رو بدم...منتهی چون عضو جدیدم نمی دونم کجا سوال بپرسم...سوالم اینه که
1=!0 قرار داده یا اثبات داره؟اگه داره چطوری اثبات می شه؟
ممنون
این یه قرار داده و قابل اثبات نیست.

SuB
23-04-2007, 15:52
سلام به تمام دوستان

یه سوال دارم اگر برام حلش کنید ممنون میشم

Y=|sin3x|+cos(tavane 2) 3x

ر ضمن کسینوس توان 2 هست

ببخشید میشه بگید چی رو باید حل کنیم؟!!!!!!
این که دادی فقط ضابطه یه تابعه. چی شو می خوای؟

mofidy1
23-04-2007, 22:13
با سلام

حد زیر را بیابید:


[ برای مشاهده لینک ، لطفا با نام کاربری خود وارد شوید یا ثبت نام کنید ]

نظرتان درباره مقدار حد زیر که حالت کلی تر حد بالاست، چیست؟



[ برای مشاهده لینک ، لطفا با نام کاربری خود وارد شوید یا ثبت نام کنید ]

موفق باشید.

ارسال متن: شنبه 25 فروردین 1386

با سلام

از دوستانی که به حل مساله پرداختند، تشکر می کنم. البته هیچکدام مساله را درست حل نکردند.

مساله را در حالت کلی تر حل می کنیم؛ یعنی ثابت می کنیم:


[ برای مشاهده لینک ، لطفا با نام کاربری خود وارد شوید یا ثبت نام کنید ]

با اضافه کردن چند جمله مناسب به سمت چپ و با توجه به اینکه


[ برای مشاهده لینک ، لطفا با نام کاربری خود وارد شوید یا ثبت نام کنید ]

داریم:


[ برای مشاهده لینک ، لطفا با نام کاربری خود وارد شوید یا ثبت نام کنید ]

که حل مساله را کامل می کند. برای حالت کلی تر، می توان همین روش را به کار برد. اما روش دیگری را که زیباتر و ساده تر است، خدمتتان تقدیم می کنم (که ایده ی آن را مدیون دوست بزرگوارم آقای مرتضی بیات - دانشجوی دکتری ریاضی مرکز تحصیلات تکمیلی در علوم پایه زنجان- هستم.)

قرار دهید:


[ برای مشاهده لینک ، لطفا با نام کاربری خود وارد شوید یا ثبت نام کنید ]

لذا


[ برای مشاهده لینک ، لطفا با نام کاربری خود وارد شوید یا ثبت نام کنید ]

بنابر این


[ برای مشاهده لینک ، لطفا با نام کاربری خود وارد شوید یا ثبت نام کنید ]

البته روش دیگری نیز بر اساس بسط تیلور وجود دارد که جای آن اینجا نیست، برای دیدن آن به لینک زیر مراجعه فرمایید:

[ برای مشاهده لینک ، لطفا با نام کاربری خود وارد شوید یا ثبت نام کنید ]

موفق باشید.

ارسال متن: دوشنبه 3 اردیبهشت 1386

mofidy1
23-04-2007, 23:25
با سلام

فرض کنید G گروهی کامل باشد( یعنی مشتق گروه برابر خود گروه باشد). ثابت کنید هر زیرگروه نرمال دوری G زیر مجموعه ای از مرکز G است.

موفق باشید.

ارسال متن: دوشنبه 3 اردیبهشت 1386

shilaa
24-04-2007, 13:35
من الگوریتم (یعنی همون روش حل) روش نیوتن و فاعده هورنر رو که ازش تو حل معادلات درس محاسبات عددی استفاده میشه رو میخوام...........



ریاضیدانان حتما میتونن جواب بدن دیگه!!!

talot
24-04-2007, 15:19
سلام من تازه اومدم هر کس میتونه هر سوالی در هر مقطعی بده

m_honarmand_j
24-04-2007, 20:29
سلام من تازه اومدم هر کس میتونه هر سوالی در هر مقطعی بده

یعنی هر سوالی باشه حل می کنید .

پاکر
25-04-2007, 00:19
از آقای مفیدی ممنونم که سوالم رو بستن ممنون!
اما دوباره میگم:
حد زیر را بدون استفاده از هم ارزی و هوپتال حل کنید:
Lim (x-sinx)/x^3 x-> 0

talot
25-04-2007, 01:25
ای بابا منظورم این بود که د رهر مقطی میتونیم سوال بپرسیم
یکی باید به من ریاضی یاد بده

new_boys333
25-04-2007, 21:11
سلام ببخشید سوالمو اینجا مطرح میکنم
استاد مدار منطقی یکی از دوستام این سوال رو مطرح کرده و گفته خودش دوارن جوونیاش 6 ماه روش فکر کرده تا جوابشو پیدا کرده حالا من میخوام از شما دوشتان این سوالو بپرسم تا شما در صورت امکان پاسخ منو بدید
سوال: 12 گلوله داریم که هیچ فرق ظاهری ندارند با هم فقط یکی از گلوله ها وزنش با بقیه فرق میکنه نه میدونیم بیشتر هست نه کمتر ، میخواهیم با 3 بار وزن کردن اون گلوله را مشخص کنیم باید چه کار کرد؟؟؟؟؟؟؟؟؟؟؟
راهنمایی به 3 گروه 4 تایی تقسیم میکنیم

ممنون